Peds (2008-2019) Flashcards

1
Q

What are 4 poor prognostic factors in a radial neck fracture in children? (2012, 2013)

A
  • Older patient (age>10y) *
  • Requires open reduction *
  • Associated Injuries *
  • Delayed treatment *
  • Internal fixation
  • Poor reduction
  • Angulation >30deg *
  • Translation >3mm *
  • *denotes presence of list in Lovell and Winter p1738
How well did you know this?
1
Not at all
2
3
4
5
Perfectly
2
Q
  1. What are 4 indications for percutaneous pinning of a distal radius fracture in the pediatric population. (2010, 2011, 2013)
A
  • Floating elbow
  • Late displacement/failure to maintain reduction in cast
  • Open fracture
  • Associated Acute compartment syndrome
  • Displaced intra-articular fracture
  • Severe soft tissue injury precluding cast

FLOADS

How well did you know this?
1
Not at all
2
3
4
5
Perfectly
3
Q
  1. What are three intra-operative techniques to reduce avascular necrosis during antegrade, locked, rigid femoral nail for an adolescent fracture? (2011, 2013)
A
  • Trochanteric/lateral start point
  • Smallest nail size
  • Minimal dissection (posterior dissection)
  • Percutaneous
  • Sharp reamers
How well did you know this?
1
Not at all
2
3
4
5
Perfectly
4
Q
  1. In a type II tibial eminence fracture (2015)
  • What is the block to reduction?
  • What are the consequences of failed reduction? (2 points)
A
  • What is the block to reduction?
    • Meniscus (usually anterior horn of medial meniscus)
    • Intermeniscal ligament
  • What are the consequences of failed reduction? (2 points)
    • ACL Instability
    • Stiffness/Notch Impingement
How well did you know this?
1
Not at all
2
3
4
5
Perfectly
5
Q
  1. 3 complications associated with pediatric tibial tubercle fracture (2012, 2014)
A

JAAOS 2002 - Fractures around the knee in children

  • Compartment Syndrome
  • Recurvatum Deformity
  • Leg Length Discrepancy
  • Hardware Irritation
  • Re-fracture
  • Stiffness
  • Patella Baja
  • Saphenous neuroma
  • Non-union
  • Skin Necrosis
How well did you know this?
1
Not at all
2
3
4
5
Perfectly
6
Q
  1. What volume of crystalloid do you bolus a peds trauma pt (in cc/kg)?
  2. 10
  3. 20
  4. 30
  5. 40
A

ANSWER: B (20cc/kg)

2009

Confirmed with ATLS manual

How well did you know this?
1
Not at all
2
3
4
5
Perfectly
7
Q
  1. 13 yr F with proximal humerus 60% displaced, 45 degrees angulated. What is the best treatment
  2. ORIF
  3. CRPP
  4. Sling
  5. Thoracobrachial cast
A

ANSWER: B (CRPP)

  • 2012
  • JAAOS - Pediatric Proximal Humerus Fractures
  • Acceptable reduction over age 10
    • <20-30o angulation
    • 50% displacement
  • Lovell and Winter p1700
    • Dobbs et al. (28) used >2/3 width displacement (Neer grade III or IV) and angulation >45 degrees in older adolescents (>12 years) as indications for attempted closed reduction under general anesthesia. If a successful reduction could be obtained (to grade II or less displacement and <45 degrees angulation), then stability was tested. If unstable, the fracture was treated with percutaneous pin fixation and immobilization. If an acceptable reduction could not be achieved closed, an open reduction was performed using a deltopectoral approach.
How well did you know this?
1
Not at all
2
3
4
5
Perfectly
8
Q
  1. 9 yo boy sustains completely displaced irreducible proximal humerus #. Intraoperatively, what is the block to reduction? REPEAT
  2. periosteum
  3. Supraspinatus
  4. LHB
  5. SHB
  6. Prism weakness
A

ANSWER: C (LHB)

  • 2010, 2012
  • JAAOS - Pediatric Proximal Humerus Fractures
    • “it should be noted that, in up to 9.4% of surgical cases, the biceps tendon can be interposed in the fracture site and may require an open incision to successfully reduce the fracture before pinning”
How well did you know this?
1
Not at all
2
3
4
5
Perfectly
9
Q
  1. 16yo female with isolated forearm fracture. You are shown an xray of patient with closed physes, anterior radial head dislocation, middle 1/3 ulna fracture and very distal ulna fracture (about 2 cm from distal ulna). What is the best treatment?

(This pic but also a distal ulna fracture)

  1. Closed reduction of ulna and radial head with a well molded above elbow cast in supination
  2. Closed reduction and intramedullary nail ulna and closed reduction of radial head
  3. Open reduction and internal fixation of ulna and radius
  4. Closed reduction and intramedullary nail fixation of ulna with closed redution of radius.
A

ANSWER: C (ORIF since closed physis = adult)

2013

  • JAAOS 1998 - Monteggia Fractures in Children and Adults
    • Transverse and short oblique fractures are adequately treated with intra-medullary wire fixation
    • Intramedullary fixation rarely provides the precise anatomic reduction that can be achieved with a plate and should not be used in adult Monteggia injuries
How well did you know this?
1
Not at all
2
3
4
5
Perfectly
10
Q
  1. Type 2 Monteggia in 10 yr old, radial head dislocated posteriorly (no x-ray). How to treat.
  2. reduce with flexion and pronation and pressure on radial head, above elbow cast in flexion and pronation
  3. reduce in flexion/supination with pressure on radial head, cast above elbow in flexion/supination
  4. reduce with extension and pressure on radial head, cast above elbow in extension
  5. open reduction with k-wire across radiocapitellar joint
A

ANSWER: C (in extension and direct post pressure)

  • 2009

Rockwood and Greens:

  • Anterior and Lateral Dislocations –> Flexion and supination for reduction
  • Posterior –> longitudinal traction, direct pressure

Lovell and Winter Page 1755

  • Treatment depends on the character of the ulnar fracture because stable, anatomic reduction of the ulna can maintain anatomic reduction of the radial head. Most Monteggia injuries in children younger than 12 years can be managed successfully by closed reduction and above-elbow casting. For a type I injury, the elbow is flexed >90 degrees with the forearm supinated. For a type II injury, the radial head may be best located in elbow extension and forearm supination (148, 149). Type III injuries are treated with a varus reduction, but are often difficult to treat with casting alone. Weekly follow-up with good quality elbow radiographs is suggested for 2 to 3 weeks to detect any recurrent radial head subluxation. Transient nerve palsies, most commonly of the posterior interosseous nerve, occur in approximately 10% of patients.
How well did you know this?
1
Not at all
2
3
4
5
Perfectly
11
Q
  1. Undisplaced lateral condyle fracture in a child. What is the next best treatment?
  2. Cast and follow up within a week
  3. Arthrogram
  4. ORIF
  5. MRI
A

ANSWER: A (Cast and f/u)

  • 2008, 2013
  • JAAOS 2011 - Lateral Condyle Fractures
    • Undisplaced fractures (<2mm)
      • Unclear prevalence: 6-69% of fractures
      • Some argue for fixation no matter what –> state that the undisplaced fracture is exceedingly rare, just an imaging problem (Flynn JPO 1989, hardacre JA JBJS 1971, Badelon O JPO 1988)
      • Some also claim they should be fixed for high rates of non-union (Flynn JPO 1989, Conner AN JBJS 1970, Speed J JBJS 1933, Fontanetta P J Trauma 1978)
  • Displaced (>2mm) or rotated
    • CRPP:
      • 2-4mm displacement
      • Avoids soft tissue stripping (non-union, osteonecrosis)
    • ORIF
      • Significantly rotated or displaced
      • Usually Kocher approach
      • Risk of non-union due to soft tissue stripping
      • No stripping posteriorly!!
      • Fixation with wires usually
      • Be careful with 3 wires –> increased loss of motion and lateral spurs
      • Charles - Dissection is usually done for you
How well did you know this?
1
Not at all
2
3
4
5
Perfectly
12
Q
  1. 6 year-old girl presents to your emergency department with a type III supracondylar fracture of the elbow. She has a cold, blue hand with no pulses. She is also unable to flex her thumb. You do your closed reduction and pinning in the operating room. When you seen her in the PACU, her hand is warm, pink, and has no pulse. She is still unable to flex her thumb. What is the best treatment at this point?
  2. Observe with close follow-up
  3. Bring back to OR for artery and nerve exploration
  4. Angio
  5. Remove K-wires
A

ANSWER: A (Observe with close f/u)

2012

JAAOS 2012 - Management of Supracondylar Humerus Fractures In Children

How well did you know this?
1
Not at all
2
3
4
5
Perfectly
13
Q
  1. A 4 yo boy has a Type III supracondylar fracture with a cold and pulseless hand. What is most true?
  2. After closed reduction, an angiogram is always needed
  3. It often takes 24 hours after reduction for a Doppler pulse to return.
  4. If a pulse doesn’t return, it can be ignored if the hand is otherwise well perfused.
  5. A complete occlusion of the brachial artery necessitates immediate reconstruction.
A

ANSWER: C (ok if warm perfused hand)

2008, 2013

Don’t like the wording here.

Robb JE (JBJS Br 2009) The pink, pulseless hand after supracondylar fracture of the humerus in children

Mangat described 19 kids with grade III treated at 6 hours from injury

11 were observed –> pulse returned at 24 hours (2), 3 weeks (3), 1-3 months (2)

How well did you know this?
1
Not at all
2
3
4
5
Perfectly
14
Q
  1. What is true regarding a type III supracondylar humerus fracture in a pediatric patient?
  2. Limited remodeling for translational deformity
  3. Splinting in 120 degrees of flexion to maintain reduction is acceptable
  4. Equivalent outcomes with lateral and crossed pinning
  5. These require emergent surgical management
A
  • ANSWER - A (limited remodelling for translational deformity) vs C (depends on wording of question on exam)
  • 2014
  • Urgent, but not emergent management
  • The Treatment of Pediatric Supracondylar Humerus Fractures J Am Acad Orthop Surg 2012;20:320-327
    • 2 lateral pins are clinically but NOT biomechanically equivalent to a crossed-pin construct - 3 lateral pins are equally strong as a crossed-pin construct; medial pin has an increased risk of ulnar nerve injury (NNH = 20)
    • Translational deformity has minimal remodelling potential and can lead to cubitus varus - the only acceptable deformity is angulation; rotation is also bad
    • Splinting in hyperflexion increases the risk of compartment syndrome 
    • The AAOS guideline suggests closed reduction with pin fixation for patients with displaced (eg, Gartland types II and III and displaced flexion) pediatric supracondylar fractures of the humerus. 
    • The AAOS guideline suggests the practitioner might use two or three laterally introduced pins to stabilize the reduction of displaced pediatric supracondylar fractures of the humerus. Considerations of potential harm indicate that the physician might avoid the use of a medial pin.

How well did you know this?
1
Not at all
2
3
4
5
Perfectly
15
Q
  1. Which of the following does not cause cubitus varus following SC #?
  2. Flexion type
  3. Inadequate reduction
  4. Loss of fixation
  5. Underappreciation of medial comminution
A

ANSWER: A (flexion type)

  • 2009
  • Flexion type supra-condylars develop cubitus VALGUS
How well did you know this?
1
Not at all
2
3
4
5
Perfectly
16
Q
  1. All of the following are true regarding a fishtail deformity of the distal humerus, except:
  2. Associated with supracondylar humerus fracture
  3. Results from central physeal growth arrest
  4. Predisposes to early ulnohumeral degenerative changes
  5. Results in significant humeral length deficiency
A
  • ANSWER: D (not true - it does not result in sig humeral length deficiency)
  • 2015
  • Glotzbecker MP (JPO 2013) Fishtail deformity of the distal humerus: a report of 15 cases
    • Proposed etiologies for fishtail deformity include avascular necrosis of traumatic and idiopathic origin and/or premature physeal arrest.
    • It can occur after displaced or minimally displaced supracondylar humerus fractures, lateral condylar fractures, physeal separations, or medial condylar fractures
    • It can also be caused iatrogenically from excessive soft tissue stripping posteriorly, posterior approaches to the lateral distal humerus, or misdirected pin passes or instrumentation placed posteriorly on the humerus.
    • Despite the fact that multiple different types of fractures may lead to this complication, the humeral deficiency seen in cases of fishtail deformity generally develops in the lateral aspect of the medial crista, trochlear groove and/or apex, which lends further support to a vascular etiology
    • In the short term, there are often minimal or no symptoms. When patients present, they may present with joint malalignment, and they often complain of pain and loss of motion secondary to joint incongruity and locking, which is related to loose body formation and joint instability.The few studies with long-term follow-up have demonstrated a high incidence of functional disability including pain and/or loss of motion.
    • Radiographically, in our series, loss of motion was associated with subluxation of the radial head. With proximal migration of the ulna, the coronoid impinges anteriorly and olecranon impinges posteriorly, leading to progressive loss of motion. Furthermore, the finding of radial head subluxation/dislocation is a radiographic sign that the proximal migration of the ulna is substantial enough to cause both loss of flexion-extension as well as radial head dislocation.
    • Joint incongruity leads to osteochondral impingement and risk of loose bodies and arthrosis.
    • Gross, I actually read this paper yesterday and thought I totally wasted my time. Glad to see there is actually a question from it!
How well did you know this?
1
Not at all
2
3
4
5
Perfectly
17
Q
  1. 12 yo boy shows up at your cast clinic 10 days after an injury with a SH II distal radius fracture. He is splinted and has 15 degrees inclination and 50 degrees apex volar angulation. What do you do?
  2. Molded short arm cast and follow up regularly
  3. Closed reduction and Molded short arm cast and follow up regularly
  4. Open reduction and pinning
  5. Open reduction and plate
A

ANSWER: A

  • 2009, 2012
  • Difficult to find evidence….pretty much assuming this is a SHII injury
  • Late re-reduction associated with damage to physis but time course difficult to sort out..7-14 days.
  • If not a physeal injury then reduction would be appropriate
  • If it is a physeal injury then ideally would do CRPP with the reduction, so I prefer A
How well did you know this?
1
Not at all
2
3
4
5
Perfectly
18
Q
  1. What is the best predictor of re-displacement in a distal radius and ulna fracture in a 7yo:
  2. Ulnar styloid fracture
  3. Cast index of 0.7
  4. Bayonet apposition
  5. Triple point index/Three point mould
A

ANSWER: D

  • 2015
  • Alemdaroglu KB (JBJS 2006) Risk Factors in Redisplacement of Distal Radius Fractures in Children
  • Kamat AS (JPO 2012) Redefining the cast index: the optimum technique to reduce re-displacement
    • “In patients with CIs of ≤ 0.8, the displacement rate was only 5.58%. However, in patients with CIs of ≥ 0.81, the displacement rate was 26%. A high CI was the sole factor that was significantly higher in the redisplacement group. No statistically significant difference was seen for age, sex, or surgeon seniority. Statistical differences were not noted in initial angular deformity or initial displacement.”
How well did you know this?
1
Not at all
2
3
4
5
Perfectly
19
Q
  1. 12 yr F with # radius. Comes back with a dystrophic looking pseudoarthrosis. What would you find on exam?
  2. hypoplastic thumb
  3. café au lait spots
  4. absent pec major
  5. Clinodactyly
A

ANSWER: B (café au lait spots )

  • 2008, 2011, 2012
  • Presumably pseudoarthrosis associated with NF1
  • Several case reports in literature
    • Kameyama O (JPO 1990), Gregg PJ (COR 1982)
  • 50% of forearm pseudarthosis are associated with NF1 (but only a small percentage of NF1 get forearm pseudarthrosis)
How well did you know this?
1
Not at all
2
3
4
5
Perfectly
20
Q
  1. What is true. Both bones forearm # 10deg rotational malunion in the forearm?
  2. A midshaft malunion will result in more decreased pronation
  3. A midshaft malunion will result in more decreased supination
  4. A distal malunion will result in more decreased pronation
  5. A distal malunion will result in more decreased supination
A

ANSWER: B (A midshaft malunion will result in more decreased supination )

  • 2009
  • Rockwood and Greens:
    • “They observed a significantly greater loss of ROM in forearms with middle-third deformities than with distal-third deformities, with more supination being lost than pronation. They also observed a significant decrease of ROM with 15° of angulation. The greater decrease of ROM in middle-third deformities was attributed to the loss of the radial bow where the two forearm bones overlap at the extremes of pronation and supination”
  • Charles’ theory: proximal to supinator insertion = supinator supinates distal fragment, = decrease supination
How well did you know this?
1
Not at all
2
3
4
5
Perfectly
21
Q
  1. What is least likely to cause compartment syndrome when putting on a hip spica cast for a femur fracture in an 8 yo
  2. Placing the below knee portion first and using it to place traction on the thigh
  3. Placing the spica in the seated position with the hip and knee at 90 degrees
  4. Including the foot in the spica
  5. Placing a spica with hip and knee flexed less than 90-90
A

ANSWER: D

  • 2013
  • Mubarak SJ (JPO 2006) Volkmann contracture and compartment syndromes after femur fractures in children treated with 90/90 spica casts
    • After child awakes from anesthesia the thigh muscles contract and the leg slips back in the cast
    • Causes pressure at the corners of the cast
  • Large TM (JBJS 2003) Compartment syndrome of the leg after treatment of a femoral fracture with an early sitting spica cast
    • “if the foot is casted then it is not available for monitoring of the compartments
    • But doesn’t cause the compartment syndrome
  • Systematic review of spica casting for the treatment of paediatric diaphyseal femur fractures (J Child Orthop 2018;12:136-144 ) - This author recommended casting the hip and knee both at 45°, not elevating the cast, and not applying traction on the below-knee cast portion during application of the cast after reduction of the fracture.
How well did you know this?
1
Not at all
2
3
4
5
Perfectly
22
Q
  1. What is the risk of osteonecrosis in a pediatric displaced trans-cervical hip fracture?
  2. 100%
  3. 60%
  4. 30%
  5. 15%
A

ANSWER: C (30%)

  • 2014
  • JAAOS 2009 - Hip Fractures in Children
    • Delbet Classification:
    • Trans-physeal Fractures
    • Trans-cervical Fractures
    • Cervicotrochanteric Fractures
    • Intertrochanteric Fractures
  • Moon (JOT 2006) Risk Factors for Avascular Necrosis after Femoral Neck Fractures in children
    • 360 cases in meta-analysis
  • Riley PM (JOT 2015) Earlier Time to Reduction Did Not Reduce Rates of Femoral Head Osteonecrosis in Pediatric Hip Fractures
    • ON Rates:
      • Type 1 - 67%
      • Type 2 - 31%
      • Type 3 - 14%
      • Type 4 - 5%
How well did you know this?
1
Not at all
2
3
4
5
Perfectly
23
Q
  1. What has increased rate of ON in pediatric femoral neck fracture?
  2. age
  3. Delbet IV
  4. fracture displacement
  5. Gender
A

ANSWER: A and C

  • 2011
  • Moon (JOT 2006) Risk Factors for Avascular Necrosis after Femoral Neck Fractures in children
    • Fracture type, displacement, age and treatment were all statistically independent predictors of AVN
    • With logistic regression analysis fracture type and age only predictors
    • Older kids 1.14x more likely to get AVN per year of age
  • Bone Joint J 2019;101-B:1160–1167.
    • We found that age (p = 0.006) and initial displacement (p = 0.001) were significant independent risk factors.
  • JAAOS 2020 - initial fracture displacement, older age, fracture type
How well did you know this?
1
Not at all
2
3
4
5
Perfectly
24
Q
  1. Pediatric patient with midshaft femoral fracture. You plan to do retrograde femur elastic nailing. What % of the canal at the isthmus should you fill with your elastic nails?
  2. 60%
  3. 70%
  4. 80%
  5. 90%
A

ANSWER: C (80%)

  • 2012
  • JAAOS - IM Nailing of pediatric femoral shaft fractures
  • Use 2 nails with combined diameter equal to 80% of narrowest diameter
  • Greater stiffness at fracture, BUT increases malreduction and rotational(posterior gapping and malalignment)
  • Each nail 40%
  • Sizes 1.5-4.0mm
How well did you know this?
1
Not at all
2
3
4
5
Perfectly
25
Q
  1. A 5yo with a femur fracture is treated with a spica cast. What is an unacceptable deformity?
  2. 25 degrees malrotation
  3. 2.5cm shortening
  4. 20 degrees coronal plane angulation
  5. 20 degrees sagittal plane angulation
A

ANSWER: B (2.5 cm shortening not acceptable)

  • 2014
  • 2009 AAOS Practice Guidelines: Treatment of Pediatric Diaphyseal Femur Fractures
    • We suggest early spica casting or traction with delayed spica casting for children age six months to five years with a diaphyseal femur fracture with less than 2 cm of shortening. Level of Evidence: II Grade of Recommendation: B
    • We are unable to recommend for or against using any specific degree of angulation or rotation as a criterion for altering the treatment plan when using the spica cast in children six months to five years of age. Level of Evidence: V Grade of Recommendation: Inconclusive
  • Management of Pediatric Femoral Shaft Fractures. JAAOS. 2004.
    • Generally, for children aged 2 to 10 years, acceptable fracture alignment at union is ≤15° of varus or valgus angulation, ≤20° of anterior or posterior angulation, and ≤30° of malrotation. Overgrowth may vary with the age of the child, the fracture pattern and location, the amount of shortening, and possibly the treatment method. In children aged 2 to 10 years, overgrowth averages 0.9 cm (range, 0.4 to 2.5 cm). Shortening at union should be no more than 1.5 cm to 2.0 cm. No more than 1.0 cm of shortening is recommended for older children.
How well did you know this?
1
Not at all
2
3
4
5
Perfectly
26
Q
  1. 54kg kid with a diaphyseal femur fracture. What is the most appropriate treatment?
  2. Ex-fix
  3. Rigid IM nail
  4. Flexible IM nail
  5. Submuscular plate
A

ANSWER: D

  • B or D depending on age of patient > 10 – rigid nail, < 10 submuscular plate
  • 2015
  • JAAOS 2011 - IM nailing of pediatric femoral shaft fracture
  • JAAOS 2012 - Submuscular plating of pediatric femur fractures
  • Flexible nails have a worse outcome with age > 11 years or weight > 50kg, consider bracing or augmenting with brace
  • If Skeletally mature, then would do rigid IM nail, otherwise sub-muscular plating
How well did you know this?
1
Not at all
2
3
4
5
Perfectly
27
Q
  1. What artery is commonly implicated in compartment syndrome in tibial tubercle fractures:
  2. Anterior tibial recurrent
  3. Peroneal artery
  4. Inferior patellar
  5. Posterior tibial recurrent
A

ANSWER: A (Anterior tibial recurrent artery)

  • 2015
  • JAAOS - Pediatric Knee Dislocations and Physeal Fractures About the Knee
  • “disruption of the anterior tibial recurrent artery can result in bleeding into the anterior compartment of the leg, leading to compression of the anterior tibial artery and deep peroneal nerve”
How well did you know this?
1
Not at all
2
3
4
5
Perfectly
28
Q
  1. In a pediatric tibial spine/eminence fracture, all of the following are true, except:
  2. ACL laxity and instability is a common complication and can cause functional impairment
  3. Meniscal and chondral injuries are not associated
  4. Associated with a larger femoral intercondylar notch
A

ANSWER: A/B

  •  JAAOS 2015 Complications of Tibial Eminence and Diaphyseal fractures in children
  • JAAOS 2010 Tibial Eminence Fractures
    • ACL Laxity (Willis RB JPO 1993)
    • 74% have laxity on KT-1000, rarely subjectively a problem
    • Laxity more prevalent in non-op patients
    • Therefore A is TRUE
  • Kocher (AJSM 2003)
    • 65% of type III have meniscal entrapment (26% type II) BUT only 3% had meniscal tears – “therefore do not appear to be commonly associated with anterior meniscal or inter-meniscal ligament entrapment”
    • Feucht MJ (KSSTA 2016)
    • Meniscal injuries in 37%
    • 30% of injuries were posterior horn longitudinal tear
    • Associated with higher age, advanced Tanner stage, pubescence
  • Kocher (JPO 2004) ACL injury vs tibial spine fracture in the skeletally immature knee
    • ACL injury had narrower notch indices than tibial spine fracture group
    • Therefore C is TRUE
How well did you know this?
1
Not at all
2
3
4
5
Perfectly
29
Q
  1. A 4 year old has a proximal tibia fracture. What deformity may occur?
  2. Varus
  3. Valgus
  4. Procurvatum
  5. Recurvatum
A

ANSWER: B (valgus)

  • 2009
  • Cozen’s fracture
How well did you know this?
1
Not at all
2
3
4
5
Perfectly
30
Q
  1. What is true regarding pediatric tibia fractures?
  2. The average 11yr old will have 10mm of overgrowth
  3. 10° of coronal displacement in an 8yr old will remodel adequately
  4. 10° of rotation is unacceptable in any age
  5. the proximal tibial physis growth will be affected even with distal fractures
A

ANSWER: B

  • 2008
  • Lovell and Winter:
  • Average overgrowth 5-7mm, almost no overgrowth in girls >8/boys >10
  • Kids <6 15o acceptable angulation, Kids > 6 10o acceptable
  • No guidelines for rotation, but won’t remodel much
How well did you know this?
1
Not at all
2
3
4
5
Perfectly
31
Q
  1. With regards to triplane fractures in children?
  2. Results from lateral rotation
  3. Occurs because posteromedial physis closes first
  4. High risk of growth arrest
  5. Cannot happen with growth plates are fully open
A

ANSWER: A (lateral rotation aka ER)

  • 2012
  • Rang’s = SER mechanism
  • Clement DA (JBJS 1987) Triplane fracture of the distal tibia. A variant in cases with an open growth plate
  • “We have reviewed 15 cases of triplane fracture of the distal tibia. The mechanism of injury is lateral rotation and the anatomical pattern of the fracture depends on the state of the growth plate at the time of injury. In seven of our cases the anteromedial part of the growth plate was fused, but in eight children the plate was completely open.”
  • “In six of these eight children there was a hump or projection of the medial growth plate. It is suggested that this hump (kump’s hump) stabilises the anteromedial part of the epiphysis in a manner similar to the partial anteromedial fusion seen in older children, and that this accounts for the occurrence of triplane fracture in the presence of an open growth plate.”
  • JAAOS triplane fractures
  • This closure proceeds from central to anteromedial to posteromedial and, finally, to the lateral portion of the epiphysis, leaving the unfused portions vulnerable to injury. The anterolateral physis is the last to close.
  • These fractures also have been termed transitional fractures. This injury usually does not occur in patients younger than age 10 years or older than age 16.7 years, but there are case reports of both.
How well did you know this?
1
Not at all
2
3
4
5
Perfectly
32
Q
  1. Which portion of the distal tibial physis is the last to fuse?
  2. Anterolateral
  3. Central
  4. Anteromedial
  5. Posteromedial
A

ANSWER: A

  • 2012
  • JAAOS triplane fractures 
  • This closure proceeds from central to anteromedial to posteromedial and, finally, to the lateral portion of the epiphysis, leaving the unfused portions vulnerable to injury. The anterolateral physis is the last to close.
How well did you know this?
1
Not at all
2
3
4
5
Perfectly
33
Q
  1. List 3 radiographic features of C2-3 pseudosubluxation (2009, 2012, 2014)
A

JAAOS 2011 - Pediatric Cervical Spine Trauma

  • Spinalaminar line within 1.5mm

AAOS Core Review:

  • Reduces with extension
  • <4mm subluxation is normal
  • No soft-tissue swelling
How well did you know this?
1
Not at all
2
3
4
5
Perfectly
34
Q
  1. List 4 ways in which radiographs of the cervical spine in children differ from those of skeletally mature patients? (2015)
A
  • AAOS Core Review 2/OKU Peds 3
    • Increased ADI (>5mm abnormal)
    • Pseudosubluxation of C2-C3
    • Loss of cervical lordosis
    • Widened retropharyngeal space (>6mm C2, >22mm at C6)
    • Wedging of cervical vertebral bodies
    • Neurocentral synchondroses (closure by age 7)
  • JAAOS Pediatric Cervical Spine Trauma:
    • Relative Horizontal Facets
    • Flat Unicate processes (
How well did you know this?
1
Not at all
2
3
4
5
Perfectly
35
Q
  1. Give 3 considerations for safe application of a pediatric Halo (2013, 2014)
A

JAAOS 2007 - Halo Fixator:

  • CT indicated in kids under 10 to determine bone thickness and rule out cranial fractures
  • Greater number of pins (10-12)
  • Torque at 2 in-lb

Miller:

6-8pins @ 2-4 inch-lbs

How well did you know this?
1
Not at all
2
3
4
5
Perfectly
36
Q
  1. 4 risk factors for SMA syndrome in peds scoliosis (2012)
A

Braun SV (JBJS 2006) SMA syndrome following spinal deformity correction

BMI < 25th percentile

<60% correction of thoracic curve on bending film

Lenke Lumbar Modifier B or C

Two stage procedure

Thoracoplasty

Combined anterior and posterior fusion

How well did you know this?
1
Not at all
2
3
4
5
Perfectly
37
Q
  1. What are the 3 components of the Lenke classification for adolescent idiopathic scoliosis? (3 lines only!) (2013)
A

Lenke LG (JBJS 2001) Adolescent idiopathic scoliosis: a new classification to determine extent of spine arthrodesis

  • Curve Type (Identify structural curves)
  • Thoracic Sagittal Modifier
  • Lumbar Modifier
How well did you know this?
1
Not at all
2
3
4
5
Perfectly
38
Q

14 yr old female with Right Thoracic curve and Left lumbar curve.

 List 3 radiographic features that are suggestive that this is a structural curve.

A

Lenke LG (JBJS 2001) Adolescent idiopathic scoliosis: a new classification to determine extent of spine arthrodesis

  • Largest Cobb Measurement
  • Side Bending Cobb >25o
  • Segmental Kyphosis > 20o
How well did you know this?
1
Not at all
2
3
4
5
Perfectly
39
Q
  1. Give 3 findings in the cervical spine in a patient with Down’s Syndrome (2011, 2014)
A

JAAOS 2006 Down’s Syndrome

  • Atlantoaxial Instability
  • Atlanto-occipital instability
  • Odontoid Hypoplasia/Os Odontoideum
  • *NO subaxial instability
How well did you know this?
1
Not at all
2
3
4
5
Perfectly
40
Q
  1. Give 3 factors to consider when performing a selective thoracic fusion in AIS (2016)
A

JAAOS 2013 Choosing Fusion Levels in Adolescent Idiopathic Scoliosis

General:

  • Preserving motion segments
  • Preventing junctional kyphosis
  • Shoulder imbalance

Or Maybe:

  • Location of Curve
  • Lumbar Modifier
  • Truncal Rotation
How well did you know this?
1
Not at all
2
3
4
5
Perfectly
41
Q
  1. List 5 radiographic findings in the cervical spine in juvenile rheumatoid arthritis (2.5pts) (2016)
A

Lovell and Winter (really distinct list - 7th Ed p880)

Radiographic Features:

  • Anterior erosion of odontoid
  • AP erosion of odontoid (apple-core)
  • Subluxation of C1 on C2
  • Focal soft tissue calcification adjacent to the ring of C1 anteriorly
  • Ankylosis of apophyseal joints
  • Growth abnormalities
  • Subluxation between C2 and C7

ASA GAFS

How well did you know this?
1
Not at all
2
3
4
5
Perfectly
42
Q

42.List 4 predictors of instability and deformity in pediatric tuberculosis of the spine (2 pts.) (2016)

A

JAAOS 2015 - Ganulomatous Vertebral Osteomyelitis’

Rajasekaran S (JBJS BR 2001) Natural history of post-tubercular kyphosis in chidlren

At Risk signs for progressive kyphotic deformity in children:

  • Gapping of the facet joints
  • Retropulsion of infected vertebra in relation to adjacent normal levels
  • Lateral listhesis
  • Toppling
How well did you know this?
1
Not at all
2
3
4
5
Perfectly
43
Q
  1. List 4 risk factors for progression in infantile scoliosis (2016)
A

JAAOS 2015 - Nonsurgical Management of Early-onset Scoliosis

  • Cobb Angle > 20
  • Intra-spinal pathology
  • Rib Phase (phase 2 rib)
  • Rib Vertebral Angle Difference > 20
How well did you know this?
1
Not at all
2
3
4
5
Perfectly
44
Q
  1. Three reasons for progression of congenital kyphosis (2012)
A

JAAOS 2004 - Congenital Scoliosis
• Failure of Formation
• Failure of Segmentation
• Mixed Deformity
Or maybe:
• Anterior unsegmented bar
• Posterior hemivertebrae
• Mixed failure of segmentation and failure of formation
• I don’t know…
• Richard also has presence of rib fusions.

How well did you know this?
1
Not at all
2
3
4
5
Perfectly
45
Q
  1. 8 organ systems associated with congenital scoliosis (2012)
A

JAAOS 2004 - Congenital Scoliosis

  • CNS - Spinal Dysraphism (18-33%)
  • GI - Anorectal Atresia
  • Pulmonary - Tracheoesophageal Fistula, Thoracic Insufficiency
  • Renal
  • Urogenital Anomalies
  • Cardiac Defects
  • Limb Defects (radial club hand/ thumb hypoplasia)
  • Auditory anomalies
How well did you know this?
1
Not at all
2
3
4
5
Perfectly
46
Q
  1. List the 4 Types of Neural Tube Defects (2010)
A
  • Spina bifida occulta
  • Meningocele
  • Myelomenigocele
  • Rachischisis
How well did you know this?
1
Not at all
2
3
4
5
Perfectly
47
Q

 47. A child with a hemivertebra. What are 3 indications for being able to perform a hemi-epiphysiodesis? (2010)

A

Hedden D (JBJS 2007) Management Themes in Congenital Scoliosis

  • Single hemi-vertebrae
  • Deformity < 50o (flexible)
  • Child < 5 years (enough remaining growth)
How well did you know this?
1
Not at all
2
3
4
5
Perfectly
48
Q
  1. What are 4 MRI findings of a patient with scoliosis and neurofibromatosis? (2011)
A

JAAOS 2010 - Orthopedic Manifestations in NF 1

MRI specific findings

  • Dural Ectasia (circumferential dilation of the thecal sac, can lead to meningocele)
  • Dumbell Lesion (neurofibroma on nerve root)
  • Parapinal masses (plexiform neurofibromatosis)

also:

  • Vertebral scalloping
  • Pencilling ribs
  • Rotation of ribs (look like twisted ribbons)
  • Enlarged vertebral foramen (from the dumbell lesion)
  • Dysplastic Pedicles
  • Spindling of the TPs
  • Short, Sharp, kyphotic curve
  • Severe rotation of apical vertebra
How well did you know this?
1
Not at all
2
3
4
5
Perfectly
49
Q
  1. What are three conditions that have dural ectasia? (2011)
A

Dural ectasia associations: (MEAN)

  • Marfan’s
  • Ehler’s Danlos
  • Achondroplasia
  • Neurofibromatosis
How well did you know this?
1
Not at all
2
3
4
5
Perfectly
50
Q
  1. List 3 spine findings in Achondroplasia. (2011)
A

JAAOS 2009 - Achondroplasia

  • Foramen Magnum Stenosis
  • Thoracolumbar Kyphosis
  • Lumbosacral hyperlordosis
  • Spinal Stenosis
  • Short, thick pedicles
  • Decreased inter-pedicular distance
How well did you know this?
1
Not at all
2
3
4
5
Perfectly
51
Q
  1. Which is true of pseudosubluxation of cervical spine in pediatric patient?
  2. Vertical facets
  3. Posterior vertebral step
  4. C3-4 subluxation is most common
  5. Intact posterior spinous line
A

ANSWER: D

2016

JAAOS 2011 - Pediatric Cervical Spine Trauma

  • Pseudosubluxation of C2 on C3 and, occasionally, C3 on C4 is the most prominent anatomic variation in the pediatric cervical spine
  • 22% incidence of C2 on C3
  • Differentiated by:
  • Swischuk line - line drawn posterior arch of C2 within 2 mm of the spinolaminar line at C1-3
  • Wedge-shaped vertebral bodies and horizontal orientation of facets may pre-dispose to injury
How well did you know this?
1
Not at all
2
3
4
5
Perfectly
52
Q
  1. “Early onset” scoliosis in a 2 yr old.. All true EXCEPT
  2. Rib overlap of the costovertebral junction (phase 2 rib) is suggestive of a progressive curve
  3. Higher chance of progression if the angle is >70°
  4. Most spontaneously resolve
  5. RVAD of 20° has a high chance of a non-progressive curve.

A

ANSWER: D

2013

JAAOS - Infantile Scoliosis

  • Spontaneously resolve in a large number of patients (12-92%)
  • Risk factors for progression:
  • Cobb Angle > 20o
  • Intra-spinal pathology (20% if Cobb > 20o)
  • Rib Phase - position of medial rib to apical vertebra
  • Phase 1 (no overlap)
  • Measure Rib Vertebra Angle Difference
  • <20o = low risk of progression
  • >20o = high risk of progression
  • Phase 2 ribs (overlap)
  • High risk of progression
How well did you know this?
1
Not at all
2
3
4
5
Perfectly
53
Q
  1. What is not a complication of VEPTR:
  2. Clavicle fracture
  3. Rib fracture
  4. Thoracic outlet syndrome
  5. Skin breakdown
A

ANSWER: A

2015

JAAOS - Growth Friendly Spine Surgery

  • Complications include wound problems, rib fractures, creeping fusion
  • Akbarnia B (Spine 2010) Complications of Growth-Sparing Surgery in Early Onset Scoliosis
  • Complications in VEPTR and Rib-Based Distraction Devices
  • Anchor problems:
  • rib fracture/cradle migration
  • Brachial Plexus problems:
  • Direct trauma or impingement if implant too lateral and cephalad
  • Compression of plexus against upper chest wall and clavicle/humerus at initial distraction and expansion
  • Chest Wall Problems:
  • Scarring and rib fusions
  • Wound Complications and Infection
  • Increased thoracic kyphosis, lumbar lordosis
  • Iatrogenic thoracic outlet syndrome secondary to vertical expandable prosthetic titanium rib expansion thoracoplasty - Journal of Pediatric Orthopedics 2008
How well did you know this?
1
Not at all
2
3
4
5
Perfectly
54
Q
  1. Congenital scoli - what is the most common CNS abnormality?
  2. Diastomyelia
  3. Chiari
  4. Tethered cord
  5. Intradural lipoma
A

ANSWER: A (old answer C)

2016

Newest evidence suggests diastematomyelia

JAAOS 2004 - Congenital Scoliosis

  • 18-37% of patients have spinal dysraphism
  • Two recent series have shown tethered cord to be most common form
  • Basu (Spine 2002) Congenital spinal deformity: A comprehensive assessment at
  • Shen J (Spine 2013) Abnormalities associated with congenital scoliosis: a retrospective study of 226 Chinese surgical cases
  • 226 congenital scoliosis, 43% intra-spinal abnormalities
  • Diastematomyelia most common
  • Ghandhari H (Eur Spine J 2015) Vertebral, rib and intraspinal anomalies in congenital scoliosis: a study of 202 Caucasians
  • Most common anomalies diastematomyelia (36%), syringomyelia (18.2%)
How well did you know this?
1
Not at all
2
3
4
5
Perfectly
55
Q
  1. All of the following are associated w/ congenital thoracic scoliosis except:
  2. Conductive hearing loss
  3. Obstructive uropathy
  4. VATER
  5. Tethered cord
A

ANSWER: A

2009

JAAOS - Congenital Scoliosis:

Anomaly may be part of VATER

“20% have existing urologic anomaly that may be asymptomatic. Renal and renal collecting system problems can vary from an asymptomatic unilateral absent kidney to obstructive uropathy”

How well did you know this?
1
Not at all
2
3
4
5
Perfectly
56
Q
  1. Juvenile scoliosis. Incidence of neuroaxis abnormalities?
  2. less than 10%
  3. 15% to 30%
  4. 45% to 60%
  5. greater than 60%
A

ANSWER: B

2011

Spine 1988 - Incidence of neural axis abnormalities in infantile and juvenile patients with spinal deformity

Juvenile incidence is 20-25%

How well did you know this?
1
Not at all
2
3
4
5
Perfectly
57
Q
  1. Highest Rate of progression for congenital Scoliosis
  2. Hemivertebrae
  3. Unilateral unsegmented Bar
  4. Double hemivertebrae
  5. Wedge
A

ANSWER: B (certainly dependent on exact question)

2012

McMaster MJ (JBJS 1982) The natural history of congential scoliosis: a study of two hundred and fifty-one patients

Block Vertebrae <2°/yr

Wedge Vertebra <2°/yr

Hemi-vertebra 2-5°/yr

Unilateral Bar 5-6°/yr

Unilateral Bar + Hemi 5-10°/yr

How well did you know this?
1
Not at all
2
3
4
5
Perfectly
58
Q
  1. Congenital kyphosis – which is true:
  2. type 2 more likely to cause neurologic problems
  3. amenable to bracing
  4. most likely cause of non-infectious paraplegia
A

ANSWER: C

2008

Lovell and Winter:

Classification:

  • Failure formation
  • Failure segmentation
  • Mixed

Neurologic risk higher with types 1 and 3

Bracing only for compensator curves

How well did you know this?
1
Not at all
2
3
4
5
Perfectly
59
Q
  1. When would you brace a congenital scoli?
  2. progressive curve 2° to unsegmented bar
  3. progressive curve 2° to hemivertebra
  4. flexible compensatory curve below congenital curve
  5. 13 decree scoli with kyphosis
A

ANSWER: C

2011

Parson-ism

JAAOS Congenital Scoliosis:

  • Primary bracing of congenital scoliosis is rarely indicated because braces do not affect progression of curve
  • Sometimes can be used to control compensatory curves
How well did you know this?
1
Not at all
2
3
4
5
Perfectly
60
Q
  1. A 5yo female with congenital scoliosis, L5 fully segmented hemivertebrae with 50 degree LS angle. What is the best treatment?
  2. L5 hemiepiphysiodesis
  3. L5 vertebrectomy
  4. Anterior and posterior fusion
  5. Posterior fusion
A

ANSWER - B

2010, 2014

JAAOS 2004 - Congenital Scoliosis

  • Anterior and posterior convex hemiepiphysiodesis is indicated in hemivertebrae when skeletally immature, but curve <40 degrees.
  • Need anterior fusion to prevent crankshaft
  • A resection of the hemivertebrae is indicated with more severe curves (>40 degrees) and younger patients (best if before age 2 - however multiples articles on ages up to 14 years).
  • Excision is recommended if balanced spine can’t be achieved with fusion alone
  • Observation only in cases of block vertebrae or wedge vertebrae which can be observed closely until skeletal maturity, as they are at lower risk of curve progression.
  • Convex Hemi-epiphysiodesis:
  • Contra-indicated in segmentation defects
  • Most patients have <15o of correction
  • Recommended in patients with curves <40o 
How well did you know this?
1
Not at all
2
3
4
5
Perfectly
61
Q
  1. Congenital scoliosis. Which is an indication for hemiepihysiodesis/ hemiarthrodesis?
  2. Age 5 to 10
  3. Unilateral bar
  4. Curve less than 50
  5. Thoracic kyphosis
A

ANSWER: C

2011

Lovell and Winter:

Results for hemi-epiphysiodesis are variable but better in kids younger than 5 with isolated lumbar hemi with progressive curve <40-50o

How well did you know this?
1
Not at all
2
3
4
5
Perfectly
62
Q
  1. Regarding SCIWORA, what is true?
  2. 50% can be a delayed presentation
  3. most common in the T spine
  4. infantile cord can stretch 2 inches before rupture
  5. most commonly seen in 8-15yo
A

ANSWER: A

2010, 2012

AAOS Core Review 2

  • Cause of paralysis in approximately 20-30% of children with injuries to the spinal cord
  • 20-50% of patients have delayed onset neurologic symptoms or late neurologic deterioration
  • Kids under age 10 are more likely to have permanent paralysis
  • JAAOS 2011 - Pediatric Cervical Spine Trauma
  • SCIWORA occurs in 18% to 38% of pediatric cervical spine injuries, and the incidence tends to be higher in young children.
  • The mechanism of injury likely relates to the flexibility of the pediatric spine, which is greater than that of the spinal cord.
  • It is believed that the spinal column can stretch up to 2 inches, whereas the spinal cord may rupture when stretched <1 cm.
How well did you know this?
1
Not at all
2
3
4
5
Perfectly
63
Q
  1. A 14yo male is post-op from a posterior spinal instrumented fusion for adolescent idiopathic scoliosis. All blood loss was meticulously replaced by the anesthetist in the OR. He has a hemovac and foley in place. He is oliguric in PACU and for 24 hours post-op despite IV at maintenance and with replacement. What is the cause of his oliguria?
  2. Excessive hemovac losses not reported by nursing
  3. Retroperitoneal hematoma
  4. Hypernatremia
  5. Decreased urine output with renal sodium excretion
A

ANSWER - D

2012, 2014

Medical complications in scoliosis surgery. Shapiro, Gary MD; Green, Daniel William MD; Fatica, Nunzia S. MD; Boachie-Adjei, Oheneba MD. Current Opinion in Pediatrics. 13(1):36-41, February 2001

Several medical complications can occur after scoliosis surgery in children and adolescents. They include the syndrome of inappropriate antidiuretic hormone; pancreatitis; cholelithiasis; superior mesenteric artery syndrome; ileus; pneumothorax; hemothorax; chylothorax; and fat embolism.

Syndrome of inappropriate antidiuretic hormone secretion in children following spinal fusion. Lieh-Lai MW; Stanitski DF; Sarnaik AP; Uy HG; Rossi NF; Simpson PM; Stanitski CL. Critical Care Medicine. 27(3):622-7, 1999 Mar.

Antidiuretic hormone (ADH) is secreted by the hypothalamus and controls water metabolism. It is released from the posterior pituitary in response to increased serum osmolality, and, to a lesser extent, a decrease in intravascular volume. In the absence of increased serum osmolality or decreased intravascular volume, the secretion of ADH in excess of the physiologic range is considered inappropriate.

Inappropriately high levels of ADH lead to excessive retention of free water and hyponatremia, which can result in fluid shifts into the intracellular fluid compartment and cerebral edema.

The elevated urinary sodium concentration associated with SIADH is thought to be caused by the action of ADH in the distal tubules, resulting in excessive reabsorption of free water and increased urinary osmolal concentration. More recently, it has been postulated that the increase in intravascular volume results in right atrial stretch, directly stimulating the release of atrial natriuretic peptide from atrial myocytes. The resultant expansion of extracellular fluid further leads to decreased proximal tubular reabsorption of sodium.

SIADH is thought to occur after spinal procedures because of stress, blood loss, invasion of the dura, and traction on the neural pathways.

Treatment for SIADH is fluid restriction and monitoring of serum and urine electrolytes and osmolality. If it is not recognized, sustained low-volume urine may be confused with hypovolemia, and increased fluid may be given. This can result in worsening hyponatremia, which can lead to pulmonary edema, cerebral edema, convulsions, coma, and death.

How well did you know this?
1
Not at all
2
3
4
5
Perfectly
64
Q
  1. Duchenne’s kid, 13 years old, scoliosis cobb angle 35 degrees, FEV 55%. Tx?
  2. wheelchair modifications
  3. bracing
  4. anterior + posterior fusion
  5. posterior only fusion
A

ANSWER - D

2012, 2014

JAAOS 2002. Duchenne Muscular Dystrophy.

In the 90% to 95% of patients with DMD who do develop scoliosis, the best treatment is early spinal fusion with internal fixation. They should be screened for the development of scoliosis by regular sitting anteroposterior spine radiographs, beginning at about age 10 years. When curvature is ascertained and reaches a Cobb angle of 20° to 30°, fusion should be done without delay.

Bracing or special seating systems may delay curve progression but will not prevent it.

there are marked disadvantages to spinal bracing and delaying surgery

First, spinal deformity is neither preventable nor responsive to nonsurgical modalities such as bracing and adaptive seating and ultimately will cause disabling deformity and severe impairment of pulmonary function… all curves in DMD progress, usually to a severe degree

sufficient spinal growth will have occurred in the child with DMD by age 10 or 11 years so that posterior fusion will not result in a marked loss of trunk height or development of crankshaft deformity.

With progression of the disease, the paraspinal muscles in these patients are progressively replaced by stiff, unyielding fibrofatty tissue, which makes the surgical dissection more difficult, reduces correctability, and increases intraoperative blood loss.

Finally, the most notable complication of surgery is postoperative pulmonary insufficiency. Patients who have a forced vital capacity of <35% (which usually occurs at age 15 years) are at high risk for pulmonary complications, whereas patients who have a forced vital capacity of >50% are likely to have no postoperative pulmonary problems and can be weaned from the ventilator the night of surgery or the morning after surgery.

Therefore, the older the patient at the time of surgery, the greater the risk of serious postoperative pulmonary complications.

For these reasons, when the curvature exceeds 20°, patients should be offered surgery.

How well did you know this?
1
Not at all
2
3
4
5
Perfectly
65
Q
  1. What is not true about achondroplasia and spine? 
  2. Atlantoaxial instability
  3. foramen magnum stenosis
  4. Lumbar Hyperlordosis
  5. Thoracolumbar kyphosis
A

ANSWER: A

2014

JAAOS 2009 - Achondroplasia: Manifestations and Treatment

Foramen magnum stenosis is associated with achondroplasia. It most commonly presents and requires treatment by age 2. It’s recommended to screen all infants with MRI or CT for foramen magnum stenosis.

Up to 80% of patients have increased limbo-sacral hyperlordosis, owing to excessive anterior pelvic tilt.

87% of patients between 1 and 2 years have thoracolumbar kyphosis. It’s is usually self-limiting as truncal muscles get stronger by walking age. Recommended to limit unsupported sitting before age 1. 

How well did you know this?
1
Not at all
2
3
4
5
Perfectly
66
Q
  1. A 12yo female presents with several months of back pain which is limiting her ability to participate in sports. After a complete history and physical exam, what is the most appropriate initial investigation?
  2. Plain radiographs
  3. SPECT-CT
  4. MRI
  5. Technetium bone scan
A

ANSWER: A

2008, 2014

JAAOS 2016 - Evaluation and Diagnosis of Back Pain in Children

Radiographs of the entire spine with the patient erect, standing position are indicated when the history or physical exam reveals localized pain, a neurologic deficit or clinical deformity

SPECT-CT sensitive and specific for spondylosis

MRI if a soft tissue problem is suspected

66% of patients have no cause identified

How well did you know this?
1
Not at all
2
3
4
5
Perfectly
67
Q
  1. All of the following are risk factors for failure of brace treatment in AIS, except:
  2. Thoracic lordosis
  3. Smoking
  4. Obesity
  5. Male
A

ANSWER: B

2015

Chiller JR (CORR 2010) Brace management in AIS

Karol LA (Spine 3002), Yrjonen (Eur Spine J 2007) - 35-38% of males uncompliant with their braces

  • Younger patients more compliant
  • It is these characteristics that cause bracing in the overweight patient to be ineffective and lead to increased curve progression compared with patients who are not overweight (Moreland M Curr Opin Orthop 1998, O’Neill PJ JBJS 2005)
  • Reference Manual for Boston Scoliosis Brace
  • Thoracic hypokyphosis (0o – 20° thoracic kyphosis) is a common feature in idiopathic scoliosis, and true thoracic lordosis (< 0° thoracic kyphosis) is probably a contraindication to bracing.
  • https://www.bostonoandp.com/Customer-Content/www/CMS/files/BostonBraceManual.pdf
How well did you know this?
1
Not at all
2
3
4
5
Perfectly
68
Q
  1. 11 yr old girl has 23 deg scoliosis. She is pre-pubertal. You send her away for 4 months to do physio. Her scoli hasn’t changed. What do you do?
  2. Boston Brace
  3. Milwaukee Brace
  4. Reassure patient / parents / FU in 4 months
  5. Posterior fusion
A

ANSWER: C

2009

I’m somewhat torn in this case. BRAIST trial used 20o as bracing cut off, but if she isn’t changing then short-interval follow up very reasonable

TLSO if apex at T7 or below

Jess thinks brace because she is going to hit her growth spurt soon. If you start noticing a change and it takes a few months to get a brace made, it might be too late… But I can see both sides. Hopefully will be more clear on real Q

How well did you know this?
1
Not at all
2
3
4
5
Perfectly
69
Q
  1. All of the following are associated with C1-2 instability, except:
  2. Achondroplasia
  3. Pseudoachondroplasia
  4. Mucopolysaccharidosis
  5. Down’s syndrome
A

ANSWER: A

2015, 2014, 2013

JAAOS 2009 Achondroplasia

Foramen magnum stenosis, thoracolumbar kyphosis, lumbosacral hyperlordosis, spinal stenosis

JAAOS 2006 - Down’s Syndrome

Atlantoaxial instability, atlanto-occipital instability, scoliosis

JAAOS - Non-traumatic upper cervical spine instability in children

  • Syndromes associated with spine instability:
  • Connective tissue –> Down’s, Marfan, Ehler-Danlos, Larsen
  • Skeletal Dysplasia –> SED, diastrophic, Kniest, Chondrodysplasia punctata, metatrophic dysplasia, mucopolysaccharidoses
How well did you know this?
1
Not at all
2
3
4
5
Perfectly
70
Q
  1. 12 yr boy with T4-T12 (Scheuermann) kyphosis of 70 degrees. What is the treatment?
  2. PT
  3. Fusion
  4. Boston Brace
  5. Milwakee Brace
A

ANSWER: D

2011

Lovell and Winter:

  • The Milwaukee brace is the brace recommended for the treatment of Scheuermann’s Disease
  • Can use Boston if apex T9 or lower
  • Indications for bracing:
  • 50-70o curve
  • Riser 0-3
  • Curve that is 40-50% correctable in brace
  • Poor prognosticators:
  • Curves > 75o
  • Wedging > 10o
  • Patient near or past skeletal maturity
  • OR:
  • Controversial indications
  • Progressive kyphosis > 75o
  • Pain not alleviated with conservative measures
How well did you know this?
1
Not at all
2
3
4
5
Perfectly
71
Q
  1. L1 chance fracture in 14 year old. Had seatbelt sign and abdo pain. Most likely injury
  2. Duodenum
  3. Spleen
  4. Liver
  5. Pancreas
A

ANSWER: A

2008

J Pediatric Surg 2001 - Abdominal injuries associated with thoracolumbar fractures after MVC

The seatbelt syndrome described in 1962 by Garrett and Branstein is characterized by the simultaneous occurrence of lumbar spine fractures and injuries to the small bowel, colon and occasionally the stomach or pancreas

How well did you know this?
1
Not at all
2
3
4
5
Perfectly
72
Q
  1. Which is NOT a risk factor for progression of scoliosis?
  2. age
  3. gender
  4. thoracic curve
  5. pain
A

ANSWER; D

2008, 2009

How well did you know this?
1
Not at all
2
3
4
5
Perfectly
73
Q
  1. Odontoid in children <7 years. All of the following except:
  2. 25% non-union
  3. Occurs at synchondrosis between C2 body and dens
  4. Displacement usually anterior
A

ANSWER: A

2009

JAAOS 2011 - Pediatric Cervical Spine Trauma

  • Typically occur through the synchondrosis of C2
  • Kelly et al. Fractures of the axis: a review of pediatric, adult, and geriatric injuries 2016.
  • More likely to happen in young <8
  • Development of C2 involves ossification of four separate ossification centers consisting of the vertebral body, the odontoid, and right and left facet/pars/lamina. The odontoid process fuses to the vertebral body between 3 and 6 years of age. A secondary ossification center develops at the apex of the odontoid process, fusing by 12 years of age
  • Fractures of the odontoid may occur through the odontoidbody synchondrosis. These fractures often heal when treated with halo or Minerva immobilization as the synchondrosis is well vascularlized
  • NO mention of non union in kids, just adults
  • https://www.ncbi.nlm.nih.gov/pmc/articles/PMC5127948/pdf/12178_2016_Article_9368.pdf
How well did you know this?
1
Not at all
2
3
4
5
Perfectly
74
Q
  1. What are 3 radiographic findings of ischemic necrosis of the femoral head following DDH treatment? (2015, 2016)
A

LaMothe Lists (Salter Criteria)

  • 1) Failure of appearance or growth of ossific nucleus at 1 year after reduction
  • 2) Broadening of femoral neck
  • 3) Increased density and fragmentation of ossified femoral head
  • 4) Residual deformity of proximal femur after reduction
  • 5) Shortening of the femoral neck
  • 6) Greater trochanter overgrowth
  • 7) Premature physeal closure
How well did you know this?
1
Not at all
2
3
4
5
Perfectly
75
Q
  1. Given an AP x-ray of a right hip (only the right hip, not a pelvis x-ray). In a 6 month old, list 4 radiographic features suggesting DDH (2016)
A

AAOS Core Review 2 p 669/ JAAOS 2000 DDH from birth to 6 months

  • Acetabular index > 25o
  • Unossified/delayed ossification of femoral heads
  • Rounding of acetabular corners
  • Disruption of Shenton’s Line
  • Not in inferomedial corner of Perkin/Hilgenreiner’s lines
  • Widened acetabular teardrop
  • Lateral center edge angle >20o (if femoral head ossified) –> weaker answer
How well did you know this?
1
Not at all
2
3
4
5
Perfectly
76
Q
  1. Unilateral DDH in 8 month old (2010)
  • list the most important clinical sign
  • list 5 possible blocks to reduction:
A

list the most important clinical sign

AAOS Core Review 2 (pg 668)

  • Range of motion (ROM) testing of the hip is important; a decrease in abduction is the most sensitive test result for DDH. ROM will be normal in children younger than 6 months however, because contractures will not yet have developed

Others: Galeazzi, LLD.

list 5 possible blocks to reduction:

AAOS Core Review p671/Vitale MG JAAOS 2001

Extra-articular:

  • Ileopsoas tendon
  • Adductor longus
  • Contracted hip capsule

Intra-capsular:

  • Inverted labrum/neolimbus
  • Pulvinar
  • Hypertrophied ligamentum teres
  • Hypertrophied TAL
How well did you know this?
1
Not at all
2
3
4
5
Perfectly
77
Q
  1. 2 radiographic risk factors for SCFE development (2012)
A

JAAOS 2006 - SCFE/AAOS Core Revire

  • Increased physeal obliquity
  • decreased neck-shaft angle
  • Decreased femoral anteversion
  • Deep acetabulum
How well did you know this?
1
Not at all
2
3
4
5
Perfectly
78
Q
  1. What are 3 endocrine conditions associated with SCFE? (2013)
A

JAAOS 2006 - SCFE/AAOS Core Review 2

  • Hypothyroidism
  • Renal osteodystrophy
  • Panhypopituitarism
  • Growth hormone deficiency treated with growth hormone
  • Hyperparathyroidism
How well did you know this?
1
Not at all
2
3
4
5
Perfectly
79
Q
  1. Give 4 indications for consideration of prophylactic pinning of the opposite hip in a child with SCFE. (2011, 2014, 2016)
A

AAOS Review:

  • Young age (open TRC, age < 10 years)
  • Endocrinopathy
  • Renal Disease
  • History of radiation
  • High risk of poor follow up (?)
How well did you know this?
1
Not at all
2
3
4
5
Perfectly
80
Q
  1. List 4 clinical or radiographic factors associated with poor prognosis in Perthes. (variations 2010, 2011, 2013, 2014)
A

JAAOS 2010 - LCP

  • Two or more Catterall “Head at Risk” (from top to bottom)
    • Femoral head subluxation lateral
    • Gage sign
    • Lateral epiphysis Calcification
    • Horizontal physis
    • Metaphyseal cysts
  • Female Gender
  • Age > 6 at presentation
  • Decreased hip ROM
  • Lateral Pillar Classification B/C, C
  • Premature physeal closure
  • Extent of subchondral fracture
  • Extent of femoral head/acetabular deformity at maturity
How well did you know this?
1
Not at all
2
3
4
5
Perfectly
81
Q
  1. Give 6 causes of acquired coxa vara in pediatric patient (2011, 2014)
A

AAOS Review

  • Perthes/AVN
  • SCFE
  • Rickets
  • Septic Arthritis/Osteomyelitis
  • Trauma
  • Fibrous Dysplasia
  • Skeletal Dysplasia
How well did you know this?
1
Not at all
2
3
4
5
Perfectly
82
Q
  1. 4 radiographic risk factors for progression of infantile Blount’s (2012)
A
  • Physeal bar across medial physis
  • Metaphyseal-Diaphyseal Angle > 16o
  • Multi-planar Deformity (varus, procurvatum, IR)
  • Beaking of proximal medial tibial metaphysis
  • Medial sloping of epiphysis
  • Irregular/widened medial physis
How well did you know this?
1
Not at all
2
3
4
5
Perfectly
83
Q
  1. 12yo girl with persistent pain and disability 6mos after an ankle sprain. Apart from fibular collateral ligament injury, what findings will you find on XR or MRI (5 points)? (2015)
A

Unclear if this is causes attributable to recurrent sprains or causes of lateral ankle pain

  • Subtle cavus foot
  • Tarsal Coalition
  • Peroneal tendon injury
  • Anterior process of calcaneus fracture
  • Lateral talar process fracture
  • Osteochondral fragment
  • Syndesmotic Ligament Injury
  • 5th metatarsal fracture
How well did you know this?
1
Not at all
2
3
4
5
Perfectly
84
Q
  1. Child with multiple ankle sprains treated non-operatively. Pain in anterolateral aspect of ankle and symptoms for 3 months. No pain with ADLs. Unable to run more than 20 minutes. You are told that he has a tarsal coalition. List 2 treatment options at this time. (2010)
A
  • Immobilization
  • Activity Modification
  • NSAIDs
How well did you know this?
1
Not at all
2
3
4
5
Perfectly
85
Q
  1. What is an Anatomic Description of CVT? (2012)
A

JAAOS 2015 - Congenital Vertical Talus

  • Dorsal dislocation of the navicular on the talar head
  • Hypoplastic and wedge shaped
  • Hindfoot is equinus and valgus
  • Midfoot and forefoot are dorsiflexed and abducted
How well did you know this?
1
Not at all
2
3
4
5
Perfectly
86
Q
  1. List 3 components of minimally invasive treatment of idiopathic congenital vertical talus. (2012, 2013)
A

Dobbs, M. B., Purcell, D. B., Nunley, R., & Morcuende, J. A. (2007). Early results of a new method of treatment for idiopathic congenital vertical talus. Surgical technique. The Journal of Bone and Joint Surgery American Volume, 89 Suppl 2 Pt.1, 111–121.

  • Serial Casting with Reverse Ponsetti Method (plantarflexion and inversion stretching)
  • Percutaneous Achilles Tenotomy
  • Percutaneous pinning of talonavicular joint
  • Boots and bars (feet at 0deg)
How well did you know this?
1
Not at all
2
3
4
5
Perfectly
87
Q
  1. 9yo boy with cerebral palsy. Ambulatory with flexible equinovarus deformity. Failed AFO and PT. Weightbearing on lateral border of foot. Persistent pain and wants to discuss operative treatment? (2015)
  • What muscles contribute to the deformity?
  • What test would help in diagnosis?
  • What operative intervention would you suggest?
A

a) What muscles contribute to the deformity?

  • Tib Ant and/ or Tib Post
  • Gastroc-soleus causes equinas

b) What test would help in diagnosis?

  • Coleman Block test
  • Confusion Test
  • Patient flexes the hip against resistance
  • Foot may show dorsiflexion by cocontraction of anterior tibialis
  • Foot will supinate (indicates that tib ant is contributing to supination during gait)
  • Can also do dynamic EMG

c) What operative intervention would you suggest?

  • TA responsible: Rancho procedure – TP lenthening, TAL, TA split trasnfer to cuboid
  • TP responsible: Split posterior tibial tendon transfer into peroneus brevis
  • Achilles/Gastroc Lengthening
How well did you know this?
1
Not at all
2
3
4
5
Perfectly
88
Q
  1. 5 yo child with right sided missed congenital hip dislocation. The BEST treatment would be:
  2. Traction followed by open reduction and innonimate osteotomy
  3. Traction followed by open reduction
  4. Open reduction with femoral shortening osteotomy and innominate osteotomy
  5. Open reduction with innonimate osteotomy
A

ANSWER: C

2013

Ryan MG (JBJS 1998) One stage treatment of congenital dislocation of the hip in children three to ten years old. Functional and radiographic results

How well did you know this?
1
Not at all
2
3
4
5
Perfectly
89
Q
  1. 3yo child, 3mos post innominate osteotomy for DDH. 30deg abduction contracture. Best treatment plan?
  2. Admit for intensive PT
  3. Femoral shortening osteotomy
  4. Traction
  5. Reassure and do nothing
A

ANSWER: D

2009, 2015

Bohm P (JBJS 2002) Salter Innominate Osteotomy for the treatment of DDH in Children

After removal of the spica cast, two other hips had a flexion and abduction contracture, which resolved after a short period of physiotherapy

How well did you know this?
1
Not at all
2
3
4
5
Perfectly
90
Q
  1. What is the most reliable method for following CP hips?
  2. Acetabular index
  3. Center edge angle
  4. Migration index
  5. Tonnis angle
A

ANSWER:C

2016

Source: Campbells

  • Children with risk factors for subluxation or dislocation should be examined and radiographs obtained at 6-month intervals until it can be established that the hips are stable, and then follow-up can be less frequent.
  • J Child Orthop. 2013 Hip Surveillance and management of the displaced hip in cerebral palsy
  • Hip dislocation in CP is potentially preventable if children are included from an early age in a surveillance programme that includes repeat radiographic and clinical examinations, and preventive treatment for hips that are displacing. A surveillance programme should be based on the child’s age, GMFCS level and migration percentage (MP), and surgical prevention may be considered in children with a MP exceeding 33 %.
  • The Reimers’ migration percentage [14] (MP) (Fig. 1) is a valid and reliable measure [13–15], and probably the most commonly used
How well did you know this?
1
Not at all
2
3
4
5
Perfectly
91
Q
  1. SCFE, all is true except
  2. Only occurs in a narrow age range
  3. Pin penetration proved to cause chondrolysis
  4. A chronic slip that has been pinned. Better to wait a few years before an osteotomy procedure.
  5. 30% in the general population and 70% of renal etiology will have a bilateral slip on initial presentation
A

ANSWER: C (poor Q)

2013

Lovell and Winter Chapter 25

  • A - TRUE
  • “most children with SCFE are pre-pubertal….average age of 12 +/-1.5 years for girls and `3.5 +/- 1.7 years for boys…At time of presentation, approximately 80% of boys are between 12-15 and 80% of girls are between 10-13”
  • The range of skeletal ages of children with SCFE has been reported to be significantly narrower than the range of their chronologic age
  • Previous years thought this was least correct
  • B - TRU
  • “Walters and Simon alerted the orthopedic community to the risk of unrecognized pin penetration in cases of SCFE treated with in situ fixation, and the associated risk of chondrolysis”
  • Millers - Associated with inadvertent pin penetration into the joint
  • C - false ??
  • Some argument that the proximal femur will remodel and may resolve the problem
  • HOWEVER, I think that if you are going to do an osteotomy procedure then you are doing it to protect the cartilage on the acetabulum so you should do it before cartilage damage occurs
  • Millers - In severe SCFE, the residual proximal femoral deformity may partially remodel with the patient’s remaining growth.
  • D - TRUE
  • 20% bilateral on initial presentation + 10-20% later development of a contra-lateral slip (not specific for renal)
  • Oppenheim WL (JPO 2003) Outcome of slipped capital femoral epiphysis in renal osteodystrophy
  • 9/11 (80%) have bilateral involvement
How well did you know this?
1
Not at all
2
3
4
5
Perfectly
92
Q
  1. SCFE what is true
  2. Two and one screw have the same risk of penetration
  3. Two and one screw have the same rate of chondrolysis
  4. Two screws has more torsional rigidity
  5. Higher risk of chondrolysis when using fully threaded screw
A

ANSWER: C

2012

JAAOS 2006 - SCFE

  • Higher risk of penetration with two screws due to more peripheral location
  • Increased penetration correlated with rates of chondrolysis
  • Single screw only has 75% of strength of two screws
  • No change in chondrolysis with full threaded…
  • Segal LS (JPO 2006) Biomechanical analysis of in situ single versus double screw fixation in non-reduced SCFE model
  • Double screw fixation was 137% stiffer than single-screw fixation under torsional loading
  • Dragoni M (JPO 2012) Biomechanical study of 1mm threaded, 32 mm threaded and fully threaded SCFE screw fixation
  • Higher rate of failure at junction of screw/threads with 16mm threads
  • No mention of chondrolysis
How well did you know this?
1
Not at all
2
3
4
5
Perfectly
93
Q
  1. SCFE; when is the child predisposed to having the screw head impinge on the labrum?
  2. 2 screws
  3. 1 screw if screw head is medial to the intertrochanteric line
  4. if screw head is distal to LT
  5. screw distal to apophysis
A

ANSWER: B

2012, 2014

  • Goodwin RC (JPO 2007) Screw head impingement after in situ fixation in moderate and severe SCFE
  • Biomechanical study
  • AP fluoroscopy revealed that screw heads lateral to the intertrochanteric line were unlikely to impinge on the acetabulum
How well did you know this?
1
Not at all
2
3
4
5
Perfectly
94
Q
  1. 9 yr Boy with Herring B (they said Herring B) hip. What is the best option for management?
  2. Observation
  3. ROM
  4. Containment surgery with either a femoral, pelvic or both
  5. Petrie casting
A

ANSWER: C

2012

Herring JA (JPO 2011) LCP disease at 100: a review of evidence-based treatment

Surgical treatment better in kids > 8, lateral pillar B, B/C (A)

How well did you know this?
1
Not at all
2
3
4
5
Perfectly
95
Q
  1. In patients with Perthes, all of these will benefit from a varus derotation osteotomy, except:
  2. 8 yo with Herring B
  3. 7 yo with lateralized/subluxed hip
  4. Epiphyseal slip angle >20% (prob index)
  5. Performing the osteotomy during initial or fragmentation phase of disease
A

ANSWER: B

2015, 2016

  • B - indicates hinge abduction, therefore a contra-indication to VRDO
  • JAAOS – SCFE
  • Intertrochanteric Osteotomy:
  • Southwick described an osteotomy for a slip 30-70o
  • Imhauser osteotomy  flexion , abduction (VALGUS), IR
  • JAAOS – Perthes:
  • Joseph B (JPO 2005)  review of 640 Perthes cases, better results with early femoral osteotomies (D)
  • Herring JA (JPO 2011) LCP disease at 100: a review of evidence-based treatment
  • Surgical treatment better in kids > 8, lateral pillar B, B/C (A)
  • ** really not the greatest evidence
  • Kamegaya M (J Child Orthop 2008) Arthrographic indicators for decision making about femoral varus osteotomy in Legg-Calve-Perthes disease
  • Epiphyseal slip-in index 20 and more associated with Herring B (lower = hinge abduction, herring C)
How well did you know this?
1
Not at all
2
3
4
5
Perfectly
96
Q
  1. Prognostic factors for Perthes - all except
  2. Age
  3. Gender
  4. ROM
  5. Extent of head involvement
A

ANSWER: C

2008

JBJS 2004 - LCP Disease. Part II: Prospective multicenter study of the effect of treatment on outcome

  • “The lateral pillar classification (p < 0.0001) and the age at the onset of the disease (p = 0.0001) were both strong prognostic factors. Female patients did significantly worse than male patients if they were over the age of 8.0 years at the onset of the disease (p = 0.004).”
  • Sagar’s/ lowell & winter also quotes reduced abduction as being poor prognostic factor
How well did you know this?
1
Not at all
2
3
4
5
Perfectly
97
Q
  1. All are true about reasons to operate for coxa vara except?
  2. HE angle > 60°
  3. Trendelenberg gait
  4. pain
  5. neck shaft of 110o
A

ANSWER: D

2011

Lovell and Winter:

Indications:

  • Symptomatic limp, Trendelenburg gait or progressive deformity
  • HE Angle > 60o
  • Progressive decrease in neck-shaft angle to 90-100o
How well did you know this?
1
Not at all
2
3
4
5
Perfectly
98
Q
  1. Which of the following does not cause snapping in the adolescent hip?
  2. Labrum
  3. Iliopsoas
  4. Rectus
  5. TFL
A

ANSWER: C

2010, 2012, 2013 (variants)

99
Q
  1. A 1 year old child is referred to your clinic with a severe LLD. Radiographs demonstrate absence of the tibia, femoral condylar hypoplasia. No active extension demonstrated. What is the best management?
  2. Fibular transfer to create a single bone lower limb
  3. Through knee amputation
  4. Symes amputation
  5. MRI to assess for residual tibial anlage
A

ANSWER: B

2016

Jones Classification/Kalamachi Classification

  • Complete absence of tibia
  • Hypoplastic lower femoral epiphysis
  • Normal femur
  • Proximal tibia intact, missing distally
  • Proximal tibia not seen
  • Diastasis of tib and fib

Clinton R (JPO 2015) Congenital Tibial Deficiency: A 37 year experience at one institution

Treatment:

  • Type 1A:
  • 85% Knee disarticulation
  • Otherwise Syme amputation with knee fusion, Brown procedures (all failed), 5 no surgery
  • 4/5 no surgery booked for knee disartic at later date
100
Q
  1. All of the following are indicated in the management of a unilateral congenital knee dislocation in a newborn EXCEPT?
  2. U/S of hips
  3. Serial casting
  4. Pavlik harness
  5. Open reduction
A

ANSWER: C

2010, 2014

  • Lots of debate - while it is clear that you cannot Pavlik a dislocated knee, the large majority reduce in 60O of flexion and can have immediate Pavlik if hip is out BUT ONLY if knee reduced first
  • JAAOS 2009 - Orthopedic Conditions in the Newborn
  • Congenital knee dislocation has ipsilateral hip dislocation in 70-100% of cases
  • Serial casting in progressive flexion
  • If concomitant DDH then treat knee dislocation first then Pavlik as knee flexion improves
  • Lovell and Winter:
  • Usually perform open reduction of knees at 6 months if needed
  • V-Y Lengthening with anterior capsular resection
101
Q
  1. A 16 month old healthy boy presents to your clinic with bowlegs. He ambulates appropriate for age, and is in the 60th percentile for his height and weight. You are shown an x-ray with varus knees, no abnormalities other than perhaps very slight beaking, metaphyseal-diaphyseal angle is measured and given to you at 12 degrees. What should you do?
  2. Observe and follow up appointment
  3. KAFO
  4. Guided growth
  5. Proximal tibial osteotomy
A

ANSWER - A

2014

JAAOS 2013 - Blount’s Disease

  • Metaphyseal-diaphyseal angle (DRENNAN ANGLE)
  • > 16 is abnormal –> 95% chance of progression
  • <10 has a 95% chance of resolution spontaneously
  • Non-operative:
  • KAFO
  • Used in kids under 3 years
  • Early stages (up to 4)
  • ~50% successful (poor evidence overall)
  • Operative:
  • Indications for surgery:
  • Age > 4
  • Langenskiold > stage III
  • Progressive deformity
102
Q
  1. 26 month old child. Tibia vara with MD angle of 19o. What do you do?
  2. Rigid KAFO
  3. Extra-articular Osteotomy
  4. Intra-articular Osteotomy
  5. Hinged Knee Brace
A

ANSWER: A

2009, 2010

  • JAAOS - Blount’s Disease
  • Bracing indicated only in patients 2-3 years of age with mild disease (Stage I-IV)
  • KAFO in kids under age 3 years
  • Indications for surgery:
  • Age > 4
  • Langenskiold > 3
  • Progressive Disease
103
Q
  1. In adolescent blounts disease, what is true about guided growth?
  2. Poor outcomes for BMI > 35
  3. Dependent on growth remaining
  4. It is associated with a high rate of hardware failure
  5. Treatment has poor results in children over age 12
A

ANSWER: B

2016

Some evidence in the JAAOS that increased BMI have increased rates of hardware failure

JAAOS 2013 – screws prone to breakage and some recommend stainless steel or non-cannulated screws

Risk factors for failure included age >14, BMI >45, severe deformity

104
Q
  1. 2 yr old treated for previous treated for idiopathic club foot with ponsetti casting. He now presents with intoeing gait and has dynamic supination with gait? What is the best method of treatment?
  2. tib ant transfer
  3. tib post transfer
  4. re cast
  5. med calc osteotomy
A

ANSWER: C

2011

  • Clubfoot: Ponsetti Management 3rd Edition (The Red Ponsetti book)
  • At the first sign of relapse, apply one to three casts to stretch the foot out and regain correction. This cast management is the same as the original Ponseti casting program. Once the deformity is corrected by casting, start the bracing program again. Even in the child with a severe recurrence, sometimes casting is very effective
  • Some children, usually between ages 3 and 4 years, with only a dynamic supination deformity will benefit from an anterior tibialis tendon transfer. This transfer is only effective if the deformity is dynamic and not fixed. Delay the procedure until after 30 months of age when the lateral cuneiform becomes ossified. Normally, bracing is not required after the transfer.
105
Q
  1. 8 yo with cavus foot and pronated 1st ray with significant symptoms for at least 3 months. Colman block test with correctable hindfoot. Best treatment
  2. Lateralalizing calcaneus osteotomy
  3. Posteromedial release
  4. 1st MT dorsiflexion osteotomy and plantar release
  5. Molded plantar orthosis
A

ANSWER: C

2013

JAAOS 2003 - Cavus deformity in children

Coleman block indicates that cavus is forefoot driven with flexible hind foot –> therefore correct first ray plantarflexion

106
Q
  1. What is an anatomic description of CVT
  2. Dorsolateral dislocation of the foot on the talus
  3. Equinus
  4. Cavovarus foot
A

ANSWER: A

2012

AAOS Comprehensive Review 2 (pg 686)

“Congenital vertical talus is an irreducible dorsal dislocation of the navicular on the talus”

107
Q
  1. What is a classic feature of CVT?
  2. calcaneus is dorsiflexed
  3. contracted Achilles tendon
  4. contracted Post tib tendon
  5. hind foot in varus, inverted
A

ANSWER: B

2011

Lovell and Winter:

  • Calcaneus and talus in equinus
  • Hindfoot in valgus
  • Tibialis posterior is subluxed over medial mal and is a dorsiflexor
108
Q
  1. What is true in congenital vertical talus?
  2. Idiopathic and syndromic cases require casting
  3. Doesn’t require TAL
  4. Pinning of the TN joint is rarely indicated
  5. The calcaneus is in dorsiflexion
A

ANSWER: A

2016

JAAOS - Congenital Vertical Talus

  • The calcaneus is in extreme equinus
  • When reduction is achieved, the patient is schedule for surgical stabilization of the TN joint with a K-wire followed by a percutaneous Achilles tenotomy
109
Q
  1. 12 yr F with painful heel. Pain worse at night, better with NSAIDS and rest. Best test to give diagnosis?
  2. MRI
  3. CT
  4. Bone scan
  5. SPECT
A

ANSWER: B

2012

Hard given the lack of information with this question, but certainly concerning for osteoid osteoma therefore CT is best option

110
Q
  1. What is not true about tarsal coalitions?
  2. MRI is best for cross sectional size estimate
  3. talo-calcaneal common
  4. best see the calcaneonavicular coalition on an oblique xray
  5. may have non-bony coalition
A

ANSWER: A

2011, 2014

Lovell and Winter:

Emery showed that MRI was nearly as good as the “gold standard CT imaging” for subtalar coalitions”

111
Q
  1. What is part of the Evans procedure for a pediatric idiopathic flexible flatfoot:
  2. Temporarily stabilize the CC and TN joints
  3. 50% increase of fusion rate with autogenous bone graft
  4. Can correct forefoot supination with opening wedge osteotomy of the medial cuneiform
  5. Osteotomy of anterior process should be 4mm proximal to the CC joint
A

ANSWER: C

2016 (2015?)

Q is probably what is commonly performed with evans procedure…cotton osteotomy (dorsal medial cuneiform opening wedge osteotomy.

JAAOS - Flatfoot Deformity in Children and Adolescents

  • The technique was interpreted, modified and further developed by Mosca, who described the strict indications for the procedures, the specific location of the osteotomy and the shape of the bone graft
  • The author described the management of the medial and lateral soft tissues, the need to temporarily stabilize the CC joint, and the need to assess and concurrently manage rigid forefoot supination deformity and contracture of the Achilles
  • Mosca JBJS
  • Osteotomy starts 1.5cm proximal to CC joint
  • Adjacent deformities become more apparent after correction of the deformity of the hindfoot. A plantar-medial opening wedge osteotomy of the medial cuneiform can be used to correct the deformity of the forefoot in skewfeet. Most longstanding flatfeet have rigid supination of the forefoot, which is a separate deformity and necessitates individual attention. A plantar medial opening wedge osteotomy of the medial cuneiform gives a satisfactory resulting many of these feet.
112
Q
  1. Have a L4 myelomeningocele, what is the foot deformity?
  2. varus deformity
  3. calcaneus deformity
  4. equinus
  5. foot valgus
A

ANSWER: A vs B (old answer)

2010, 2011

  • As per Goldstein
  • Can go either varus or valgus but the real problem is the calcaneus
  • Varus/valgus can be braced, calcaneus cannot
  • From OKU Peds 4 Pg. 110 and OKU 10 p. 817 (old answer)
  • 50% of kids with myelodysplasia have a foot deformity
  • Talipes equinovarus (rigid clubfoot), Calcaneus, Calcaneovalgus, Calcaneovarus and Vertical Talus are the most common deformities
  • Note that the level of the cord lesion is not always predictive of the foot deformity
  • Calcaneus Deformity
  • Results most commonly with L4 involvement, but can occur with L5.
  • L4 level creates weakness of the gastro soleus and spasticity of the tib ant, which leads to the calcaneus deformity.
  • The tib ant dorsiflexes the ankle and supinates the forefoot. This does not mean it causes cavus. It means it causes calcaneus (+/- valgus depending on if the peroneals are working, which is more common if its L5 not L4). See picture below.
  • Creates a crouch gait
  • Surgery includes achilles tenodesis and tib ant transfer to the heel to try and correct the deformity, which occurs b/c the tib ant over powers the non functioning gastro-soleus complex
  • Calcaneovalgus deformity
  • Occurs with L5 level most commonly, but can occur with L4 level as well
  • Results in spasticity of the foot DF and everters (tib ant and peroneals, L4 and L5 nerve roots) with weakness of the gastroc soleus and tib post (the plantarflexors and inverters, S1 and S2 nerve roots).
  • Creates unopposed DF and eversion, leading to the deformity
  • Whether a calcaneus or a calcaneovalgus foot deformity develops depends on if the peroneals are working.
  • Clubfoot deformity (Equino-cavo-varus)
  • This is the most common deformity
  • Results from contractures of the intrinsic muscles of the foot and usually occurs with high lumbar and thoracic level myelodysplasia
  • Treatment is with ponsetti method, but there is a high failure rate after 1st course of treatment (60%), most of which will correct with a second round of ponsetti casting and achilles tenotomy. The benefit of using casting for this is that you avoid large open releases, which have high complication rates.
  • Insensate feet have higher complication rate than sensate feet with ponsetti casting and need close follow up

113
Q
  1. Kid with pes cavus foot. Hindfoot varus corrects with coleman block test. What is the best management? 
  2. Plantar fascia and 1st MT osteotomy
  3. Calcaneal osteotomy and FDL transfer
  4. Observe
  5. Orthosis – as initial management
A

ANSWER: D

2013

SIGH.

If NORMAL kid and mild deformity –> non-op

If NEURO kid –> Op

114
Q
  1. What is the main advantage of a closing wedge cuboid osteotomy over lateral calcaneal slide for cavus foot with metatarsus adductus? (2008)
  2. Higher rate of union
  3. Better cosmesis
  4. Better correction of forefoot deformity
A

ANSWER: C

Better correction of hindfoot deformity 

115
Q
  1. Residual deformity after clubfoot treatment. Which has the worst functional outcome? (2009)
  2. Weak gastroc / residual hindfoot deformity
  3. Hindfoot varus / forefoot adductus
  4. Forefoot adductus
  5. Residual hindfoot deformity / equinus contracture
A

ANSWER: A

116
Q
  1. Adult with treatment of foot condition with casting and surgery. Now has flat top talus. What was the condition: (2008)
  2. TEV (Clubfoot)
  3. CVT
A

ANSWER: A 

117
Q
  1. 12 y.o. with diastomatomyelia and an associated calcaneovalgus foot. What is the treatment? (2008)
  2. Non-op, aggressive physio and AFO
  3. Peroneus brevis to longus transfer
  4. Tib Ant to calcaneus transfer
  5. Split posterior tibialis transfer
A

ANSWER: A

118
Q
  1. Which osteotomy is contraindicated in an ambulatory child with a dislocated hip secondary to a sacral level Myelo. (2013)
  2. Dega
  3. Pemberton
  4. Chiari
  5. Salter
A

ANSWER: D

119
Q
  1. All of the following pelvic osteotomies have normal articular cartilage articulating with head except (2013)
  2. Dega
  3. Bernase PAO
  4. Chiari
  5. Salter
A

ANSWER: C

120
Q
  1. Order of ossification of pediatric elbow (2012)
A

CRITOE

  • Capitellum –1
  • Radial head – 3
  • Internal (med) epicondyle – 5
  • Trochlea – 7
  • Olecranon – 9
  • External (lat) epicondyle - 11
121
Q
  1. Regarding syndactyly, define the following terms (2015)
  • · Simple
  • · Complex
  • · Complicated
  • · Complete
  • · Incomplete
A

· Simple – web formed by soft tissue only

· Complex – fusion of adjacent phalanges

· Complicated – interposition of accessory phalanges or abnormal bones

· Complete – webbing extends to tips

· Incomplete – does not extend to tips

122
Q
  1. List 3 x-ray findings of Madelung’s deformity.(2011)
A

· Vicker’s ligament (tethers the lunate to the radius)

· Increased radial inclination

· Volar subluxation of the carpus

· Apparent dorsal subluxation of the ulna

123
Q
  1. Four Clinical Findings that would suggest posterior shoulder dislocation in a six month old with a brachial plexus palsy? (2014)
A
  • o In setting of persistent muscular imbalance (IR contracture – loss of ER) across the developing shoulder, there is progressive dysplasia of the glenohumeral joint (Glenoid hypoplasia), with posterior subluxation of the humeral head, humeral head flattening, and increased glenoid retroversion – diagnosed on ultrasound
  • Internal rotation contracture (i.e. loss of ER)
  • Limited FE or ABD
  • Mild axillary webbing
  • MDI
124
Q
  1. Syndactyly. What is true? (2013, 2016)
  2. Complex syndactyly means that they share a common nerve
  3. Functional improvement is an indication for surgery
  4. There is no possible surgery to fix the fused nail
  5. Skin bridge goes to the PIP level
A

ANSWER: B

125
Q
  1. What is true about Sprengel (2008)
  2. Does not make a cosmetic or functional difference
  3. Can do up to age 12
  4. Clavicular osteotomy decreases incidence of neuro injury
  5. Does not makes a difference in abduction
A

ANSWER: C

Patients 3-8 years old (>8 increased risk NV injury – they then need a clavicle osteotomy)

126
Q
  1. Which of the following is most true regarding neonatal brachial plexus injury? (2013)
    a. Caesarian section eliminates the risk
    b. Lack of biceps flexion return by 6 months is suggestive of neurotemesis
    c. Most resolve without surgery
    d. Lower plexus injuries (Klumpke’s) are more common
A

ANSWER: C

o 80% to 90% of patients demonstrate spontaneous recovery.

o If antigravity biceps function recovers by 2 months, full recovery is anticipated.

o If biceps function recovers at or after 5 months, incomplete recovery is likely.

o Erb’s – more common, better prognosis

127
Q

127 .Which of the following is true of birth brachial plexus palsy (2014)

a. Phrenic nerve injury is indicative of root avulsion
b. Horner’s usually associated with C5 injury
c. No biceps at 3 months indicative of neurotemesis
d. Neurotmesis is usually repairable

A

ANSWER: A

o Presence of Horner syndrome (constricted pupil – miosis, lazy eye – ptosis, and anhidrosis) portends worse prognosis (<10% recovery)

§ Other signs of preganglionic lesion

· Elevated hemidiaphragm

· Winged scapula

· Absence of rhomboid/rotator cuff/lat dorsi function

128
Q
  1. What is not correct regarding obstetrical brachial plexus injury: (2015)
    a. Should perform a primary nerve repair in Erb’s palsy within 3 months if no return of biceps function
    b. Decreased ROM of the shoulder will eventually lead to Xray changes
    c. De-rotational osteotomy of the forearm and proximal humerus are treatment options in some cases
    d. External rotators of the shoulder are weaker than the internal rotators
A

ANSWER: A

· Procedures for Erb’s Palsy

o Can trial splinting – SupER splint (supination, ER splint)

o Pectoralis major release

o Subscapularis release

o Anterior capsule release

o Latissimus & teres major transfer: for younger children to prevent/stabilize dysplasia. Do it with the anterior releases

o External rotation osteotomy: for older children with significant dysplasia

129
Q
  1. 7 year old with obstetrical brachial plexus injury and has posterior glenohumeral dislocation. What gives best outcome now (2008)
    a. Subscap lengthening with capsular release
    b. Humerus derotational osteotomy
    c. Anterior open reduction with posterior capsular plication
    d. Neglect
A

ANSWER: B

o Humeral osteotomy: external rotation osteotomy of the distal humeral segment; recommended in setting of advanced glenohumeral joint dysplasia

§ Performed > 5 years of age

130
Q
  1. Clinical exam of a boy with CP and intrinsic contracture would reveal which of the following?(2008, 2013)
    a. MCPs extended, PIPs flexed
    b. MCPs extended, PIPs extended
    c. MCPs flexed, PIPs extended
A

ANSWER: C

131
Q
  1. Pt. With CP upper extremity with thumb in palm deformity. All are true except? (2011)
    a. Web space contracture
    b. Spastic adductor
    c. Extensor of EDB
    d. Stiff MCP joint
A

ANSWER: D

132
Q
  1. A patient shows up in your office with a congenital radial head dislocation. She is skeletally immature. What is true about treatment with a radial head excision? (2016)
    a. She will develop cubitus valgus
    b. Her radius will migrate proximally
    c. She is likely to develop a radioulnar syonstosis
    d. The radial head will regrow
A

ANSWER: B

133
Q
  1. Radiographic findings in OCD elbow in an 11year old female gymnast (2009)
    a. Radial Head Enlargement
    b. Fragmentation of trochlea
    c. Fragmentation of radial head
    d. Late physeal closure
A

ANSWER: A

· XR

o Greenspan 45 degree view

o Fragmented subchondral bone with lucencies and irregular ossification of the capitellum

§ In Panner’s capitellum is fragmented and smaller and involves entire capitellum (whereas OCD is typically only anterior)

§ Rarely, may see radial head changes / OCD

o Intra-articular loose bodies – more common with OCD

o Enlarged radial head (appears large)

o Image contralateral side for comparison to identify subtle changes

134
Q
  1. Kid with hypoplastic thumb and unstable MCP and CMC joint. What to do for definitive treatment? REPEAT (2008, 2012, 2013)
    a. Opponenplasty
    b. Index pollicization
    c. Splint
    d. Amputate
A

ANSWER: B

· Hypoplastic thumb, unstable MCP + CMC = type 3B

135
Q
  1. Congenital pseudoarthrosis of clavicle. Which of these is NOT true? (2016)
    a. Edges of pseudoarthrosis form physis-like end
    b. Synovial fluid found between ends
    c. Something about the position/relationship of the 2 ends
    d. Always at mid-clavicular region with larger sternal end that is superior, anterior translation relative to the acromial segment
A

ANSWER: B

  • · The radiographic findings are characteristic and include clear separation in the middle portion, with the medial fragment positioned above the lateral fragment because of the action of muscle forces and the postural traction exerted by the weight of the upper limb
  • Hirata et al(18) demonstrated that the chondrocytes at different stages of maturation of the extremities of the pseudarthrosis presented a columnar layout, similar to the patterns seen in epiphyseal growth plates.
136
Q
  1. What is not associated with radial club hand? (2011, 2014)
    a. hypoplastic thumb
    b. thrombocytopenia
    c. cardiac defect
    d. elbow instability
A

ANSWER: D

· Thumb and index finger Hypoplasia (80%)

· Longitudinal failure of formation of radial side of forearm, wrist and hand

· Associated with:

o TAR – low platelet count, normalizes over time

o Fanconi’s Anemia - Platelet and blood cell counts normal at birth but decrease dramatically during first few years of life; diagnosed with mitomycin-C chromosomal challenge test; treated with bone marrow transplantation

o Holt Oram - RLD with congenital heart disease, typically atrial or ventricular septal defects

o VACTERL

o Craniofacial anomalies

137
Q
  1. All of these are true regarding congenital trigger thumb, except: *REPEAT (2011, 2014, 2016)
    a. Also called congenital clasp thumb
    b. Rarely bilateral
    c. Rarely triggers
    d. The tendon sheath is normal
A

ANSWER: A (B also)

138
Q
  1. All of the following are associated with short metacarpals except: (2015)
    a. Congenital glaucoma
    b. Acrosyndactyly (McGill)
    c. Hypoparathyroidism
    d. Sickle cell anemia
A

Answer: B

Common causes 2:

· idiopathic

· post-infective (e.g. osteomyelitis, yaws, tuberculosis dactylitis)

· pseudohypoparathyroidism/pseudopseudohypoparathyroidism

· post-traumatic (acute or chronic/healed and particularly those involving growth plate)

· Turner syndrome

Uncommon causes 2:

· basal cell nevus syndrome (Gorlin syndrome)

· hereditary multiple exostosis syndrome 5

· juvenile idiopathic arthritis

· sickle cell disease with secondary infarction

· homocystinuria

· Langer-Giedion syndrome

Glaucoma + short MC - Weill-Marchesani syndrome is a rare connective tissue disorder characterized by microspherophakia, severe myopia, acute and/or chronic glaucoma, and cataract. Other features include brachydactyly and short stature.

139
Q
  1. What is true regarding pediatric scaphoid fractures: (2015)
    a. 2/3 are distal pole fractures
    b. Usually undisplaced waist fracture
    c. Proximal pole fractures are common
    d. Account for 5% of upper extremity fractures in children
A

ANSWER: A

· Scaphoid fractures are most commonly treated with a thumb spica cast. There is no consensus regarding short- or long-arm thumb spica use.

o Scaphoid matures radiographically from 6-15 years

o Fracture of distal 1/3rd most common in children

§ Also: avulsions, tubercle fracture, waist fracture, proximal pole fracture

o Distal pole / small avulsion fractures can be treated in short arm spica

· Distal pole fractures routinely heal with closed treatment

· Waist fractures (especially in adolescents) have worse results and may result in osteonecrosis and/or nonunion.

TFCC tears may be seen with distal radial and/or ulnar styloid fractures and are generally treated closed

140
Q
  1. 11 yo female gymnast with wrist pain - what would you expect to find on XRAY? (2015)
    a. Positive ulnar variance
    b. Decreased radial inclination
    c. Negative ulnar variance
    d. CMC arthritis
A

ANSWER: C (changed from A)

JAAOS 2019- Artistic Gymnastics Injuries; Epidemiology, Evaluation, and Treatment

Unlike an adult wrist, the immature wrist typically exhibits negative ulnar variance, resulting in a higher distribution of load to the immature distal radius, especially during high compressive forces. Chronic compression can lead to growth arrest of the physis, causing premature closure of the ulnar aspect. A deformity results when the distal radius is shifted ulnar and volar.6 ,7 Risk factors include the female sex, and repetitive activities with extreme wrist dorsiflexion.

Immature wrist = ulnar negative…..with prolonged load, can have growth arrest at DR = ulnar positive

· Distal radius epiphysiolysis/epiphysitis

o Injury to the distal radial epiphysis most commonly occurs in adolescent athletes in sports that require weight bearing on the upper extremities, such as gymnastics or cheerleading

· Radiographic findings

o Widened physis

o Blurred growth plate

o Metaphyseal changes

o Fragmentation of radial and volar physis

o Long term à Positive ulnar variance (TFCC/ulnar abutment)

141
Q
  1. 12yo bilateral camptodactyly how treat (2008)
    a. Volar capsulotomy
    b. Splinting
    c. Tendon transfers
    d. Fusion
A

ANSWER: B

Nonoperative

  • Nonoperative treatment is favored in most cases
    • Indications
      • Best for PIP contracture < 30 degrees
    • Technique
      • Passive stretching + static splinting
    • Outcomes
      • Variable outcomes
      • Best outcomes with early intervention

Operative

  • Indications
    • Progressive deformity leading to functional impairment
  • Technique
    • Must address all abnormal anatomy
    • Passive (correctable) deformities
      • FDS tenotomy, or
      • FDS transfer to radial lateral band if full active PIP extension can be achieved with MCP flexion
      • Osteotomy vs. arthrodesis
        • Indications
          • Severe fixed deformities
    • Outcomes
      • Variable outcomes
142
Q
  1. Kid with big index finger with DIP and PIP stiffness, lipofibromatosis, gets in way of pinch between thumb and middle (young kid with a macrodactyly and associated finger stiffness) (2008)
    a. index ray amp
    b. debulk
    c. epiphyseodesis
    d. liposuction
A

ANSWER: A

143
Q
  1. List the 6 orthopaedic findings for diagnosis of Marfan’s. (2015)
A
  • Arachnodactyly
  • Scoliosis
  • Acetabular protrusion
  • Patella dislocation
  • ligamentous laxity
  • pes planovalgus
  • dural ectasia
  • pectus excavatum/carinatum
144
Q
  1. List 3 xtraskeletal features of Marfan’s syndrome. (2013, 2010)
A
  • Aortic root dilatation
  • Mitral valve prolapse
  • Spontaneous pneumothorax
  • Lens dislocation (superior)

LAMS

or SMAL

145
Q
  1. What are 4 risk factors for the development of DVT associated with Methicillin resistant staphylococcus aureus pediatric osteomyelitis? (2013)
A
  • Panton-Valentine Leukocidin
  • CRP>6
  • Age >8
  • Surgical tx
146
Q
  1. Except for joint aspiration, what are 4 clinical signs that are suggestive of septic arthritis from transient synovitis. (2010, 2011, 2013, 2014)
A
  • WBC > 12
  • ESR > 40
  • Temp > 38.5
  • NWB status
  • = Kocher criteria*
147
Q
  1. List 4 long term complications of pediatric osteomyelitis? (2011, 2016)
A
  • LLD
  • Growth arrest
  • Angular deformity
  • Overgrowth

GAOL

148
Q
  1. List 3 benefits to using steroids (prednisone/deflazacort) in Duchenne’s muscular dystrophy?
A

Prolong mobilization, slows progression to scoliosis, improves resp function

· Prolongs ambulation

· Slows progression of scoliosis

· Slows deterioration of Forced Vital Capacity (FVC) (because of delay in scoliosis)

· Slows deterioration in cardiac function (because of delay in scoliosis)

149
Q
  1. List 3 potential complications of using steroids in Duchenne’s muscular dystrophy?
A

· AVN

· Weight gain

· Cushingoid

· GI symptoms

· Mood swings

· Short stature

· Cataracts

· Insufficiency fractures - spine

150
Q
  1. 12 yr old female with a 30 degree thoracic curve. List 6 ways to determine her skeletal maturity. (2010, 2014)
A

· Iliac crest apophysis – Risser grade

· Olecranon apophysis

· Triradiate – open or closed

· PA hand (bone age)

· Menses - Represents end of peak height velocity – beginning of slowing growth

· Tanner stage

· Peak growth velocity

151
Q
  1. 9yo boy with cerebral palsy. Ambulatory with flexible equinovarus deformity. Failed AFO and PT. Weightbearing on lateral border of foot. Persistent pain and wants to discuss operative treatment? (2015)
    a) What muscles contribute to the deformity?
    b) What test would help in diagnosis?
    c) What operative intervention would you suggest?
A

a) What muscles contribute to the deformity?

· Tib Ant, Tib Post

gastrocs, too

b) What test would help in diagnosis?

· Coleman Block test

· Confusion Test

§ Helps distinguish AT vs PT as primary muscle

§ Patient flexes the hip against resistance

§ Foot may show dorsiflexion by cocontraction of anterior tibialis

§ Foot will supinate (indicates that tib ant is contributing to the equinovarus deformity)

c) What operative intervention would you suggest?

· TA main – split TA transfer, TP main - Split posterior tibial tendon transfer into peroneus brevis

· Achilles/Gastroc Lengthening

152
Q
  1. List 3 factors to consider when thinking about doing a hemi-epiphyseodesis (2010)
A
  • Amount of growth remaining
  • Amount of correction needed
  • Contralateral limb
153
Q
  1. 2 week old with effusion and ultrasound showing effusion and not moving arm. What is the most common organism (2010)

a. Staph aureus
b. GBS
c. Hib
d. Neisseria meningitis

A

ANSWER: B

NEONATES

Home = GBS

NICU = S. aureus

JAAOS says s aureus most common overall? But mentions normal kid with tranvaginal delivery that goes home and comes back with septic joint = GBS

154
Q
  1. Child with arthrogrypotic extension contracture. All of the following are possible to restore elbow flexion except: (2016)

a. Triceps
b. Steindler flexorplasty
c. Pec Major
d. Anterior deltoid

A

ANSWER: D

Tendon transfer for elbow flexion – triceps, Steindler flexorplasty (move FP mass proximally on humerus), pec major tx

155
Q
  1. Regarding the Stindler flexorplasty in arthrogryposis, which is not correct : (2008)
    a. Techniques are described for proximal attachment to both bone and soft tissue
    b. It is associated with a 20% loss of flexion power (they didn’t say wrist or elbow)
    c. It results in elbow flexion strength being stronger in supination than pronation
A

ANSWER: C

Steindler flexorplasty (move FP mass proximally on humerus),

o Since using FP mass – may be stronger in pronation?

156
Q
  1. Regarding arthrogryposis, all are true except? (2015)
    a. bilateral hip dislocation
    b. normal intelligence
    c. progression of flexion contracture until skeletal maturity
    d. internal rotation contracture of shoulder
A

ANSWER: C

157
Q
  1. All of the following are true, except: (2013, 2015)
    a. Mucopolysaccharidosis is a proportionate form of dwarfism
    b. San Fillipo is the most common form of mucopolysaccharidosis
    c. Duchenne’s is associated with calf hypertrophy
    d. Marfan’s is associated with arachnodactyly
A

ANSWER: C

158
Q
  1. All of the following are associated with Marfan’s syndrome, except: (2016)
    a. Type III Collagen
    b. Scoliosis
    c. Aortic dilatation
    d. Protrusio acetubuli
A

ANSWER: A

159
Q
  1. All of the following about neurofibromatosis scoliosis are true except: (2013)
    a. In a dystrophic curve, the Cobb angle is not predictive of progression
    b. Dystrophic curves are most common
    c. If scoliosis presents younger than age 8, 70% will become dystrophic
    d. Associated with dural ectasia
A

ANSWER: B

  • JAAOS 2010 – Orthopaedic Manifestations of NF1
  • Dystrophic curves are LESS common

o Scoliosis is common in patients with NF

o Nondystrophic scoliosis in NF is treated like adolescent idiopathic scoliosis

o Dystrophic scoliosis is short (4 to 6 levels), with sharp curves, and often occurs in children younger than age 6 years

§ Characterized by scalloping end plates (posteriorly most common), foraminal enlargement, and penciling of ribs (narrower than 2nd rib), vertebral wedging, paravertebral soft tissue mass, widened interpedicular distance, dysplastic pedicles, dural ectasia, dumbbell lesion (canal neurofibroma)

§ A combined kyphoscoliosis progresses more rapidly, as well as having 3 or more of the above osseous feature

· Type 1 – kyphosis < 50 degrees

· Type 2 – kyphosis > 50 degrees

§ 87% of curves rapidly progress when three or more ribs are penciled.

§ Dystrophic scoliosis in NF is resistant to brace treatment.

§ Dystrophic scoliosis in NF is treated with early anterior and posterior fusion (>20 degrees)

160
Q
  1. Conditions associated with NF-1; all except (2012)
    a. Hypertension
    b. Malignant CNS tumor
    c. Short stature
    d. Acoustic Neuroma
A

ANSWER: D

· Extremity deformities

· Congenital bowing and pseudoarthrosis of tibia and forearm

· Hemihypertrophy – seen with plexiform NF

· Neoplasias

o Plexiform are larger, and have a vascular supply; can transform into malignant nerve sheath tumor (MPNST)

o These are very aggressive tumors, high rate of recurrence

· HTN, stroke, congenital heart disease

· Neurofibromas

· Scoliosis

o Note: before scoliosis OR – get MRI to rule out dural ectasia and intraspinal neurofibromas

o Etiology à osteomalacia, intraspinal neurofibroma

· Kyphosis

· Atlantoaxial instability – higher rate with dystrophic curves

· Reduced bone mass – metabolic bone disorder

Acoustic neuroma = schwanomma = NF2!

161
Q
  1. Which of the following factors is NOT true regarding NF-1? (2014, 2016)
    a. If father has the gene, worse for patient than if mother has the gene
    b. 50% sporadic mutation
    c. 6% tibial pseudarthrosis rate
    d. short stature
A

ANSWER: A

· Autosomal dominant

o Spontaneous mutations make up 50% of cases

o Complete penetrance of NF1 gene

162
Q
  1. Proteus syndrome. What is associated? (2009)
    a. Megalospondylodysplasia
    b. Café au lait
    c. Lisch Nodules
    d. Precocious puberty
A

ANSWER: A

3 characteristic findings

  • hemihypertrophy
  • macrodactyly
  • partial gigantism of hands/feet/both
163
Q
  1. What is true regarding achondroplasia? (2011)
    a. point mutation of FGFR3
    b. hypertrophic zone is normal
    c. 50 % are spontaneous presentations
    d. autosomal recessive
A

ANSWER: A

o Results in growth retardation of proliferative zone (hypertrophic zone also abnormal)

164
Q
  1. Achondroplasia, all of the following except (2008)
    a. Foramen magnum stenosis
    b. X-linked
    c. Most common short limbed disproportionate dwarfism
    d. FGFR3
A

ANSWER: B

165
Q
  1. Osteogenesis Imperfecta, what is the least true: (2013)
    a. Disease symptoms get better as a patient ages.
    b. Healed fractures have abnormal strength
    c. Heal bones slower
    d. Over 70% develop scoliosis
A

ANSWER: C

OI – DOESN’T take longer to heal

number of fractures typically decreases as patient ages and usually stops after puberty

166
Q
  1. 6 yr old boy with distal radius # after minor trauma seen in clinic. Pt has had 4 #s with minor trauma in the last 12 months. Also has bowing of legs. What is the defect? (2011)
    a. defect in collagen 1
    b. defect in collagen 2
    c. defect in phosphate
    d. child abuse
A

ANSWER: A

167
Q
  1. What is not a side effect of bisphosphonate treatment in kids (2013)
    a. Acute fever with administration
    b. Growth delay
    c. Prolonged effects on bone remodelling
    d. Immediate and transient hypocalcemia
A

ANSWER: B

ANSWER: B

  • https://www.ncbi.nlm.nih.gov/pmc/articles/PMC4279811/pdf/40200_2014_Article_109.pdf
  • Children with osteogenesis imperfecta treated with bisphosphonates at early age have normal or improved growth and new bone acquisition is reported in studies on long bone fractures after discontinuation of pamidronate therapy
  • In most cases acute phase reaction is observed with fever, malaise, abdominal pain, vomiting, muscle or bone pain with the initiation of either intravenous or oral agents within 1 3 days, and lasting few days [1,28]. Asymptomatic hypophosphatemia, and hypomagnesaemia and hypocalcemia causing tetany are rare
  • The greatest concern in the young patients is long term suppression of bone turnover.
168
Q
  1. Hypophosphatasia. Which of the following are found in a patient with hypophosphatasia? (2013)
    a. Low ALP, Normal Ca, Normal PTH
    b. Low ALP, High Ca Normal PTH
    c. High ALP, Low Ca, Normal PTH
    d. High ALP, Normal Calcium, High Cocaine
A

ANSWER: B

  • Labs
  • serum
  • decreased serum alkaline phosphatase that converts phosphate to inorganic phosphate for bone mineralization
  • Decrease ALP, increase Ca, normal PTH
  • urine
  • phosphoethanolamine in the urine diagnostic for hypophosphatasia
169
Q
  1. Osteomalacia causes which of the following (2013)
    a. Increased mineralization after fracture
    b. Pseudofractures
    c. Biopsy shows increased mineralized osteoid matrix.
    d. Sasquatches have weak bone
A

ANSWER: B

170
Q
  1. Duchenne muscular dystrophy inheritance pattern (2013)
    a. Y linked
    b. AR
    c. X linked
    d. AD
A

ANSWER: C

171
Q
  1. . Duchenne MD - All Except: (2008)
    a. Often present with toe walking
    b. Apparent at birth
    c. X-linked
    d. Proximal muscle weakness
A

ANSWER: B

172
Q
  1. All of the following are true regarding the use of steroids in Duchenne’s muscular dystrophy, except? (2008, 2016)
    a. Improves muscle strength
    b. Prolongs ambulation
    c. Does not improve pulmonary function
    d. Osteopenia is a concern
A

ANSWER: C

· Prolongs ambulation

· Slows progression of scoliosis

· Slows deterioration of Forced Vital Capacity (FVC) (because of delay in scoliosis)

· Slows deterioration in cardiac function (because of delay in scoliosis)

173
Q
  1. CP kid which is not a good prognostic factor (2008)
    a. hand head to knee in 5 sec
    b. IQ >90
    c. ability to differentiate textures
    d. 2 point <10mm
A

ANSWER: D

174
Q
  1. Who is a poor Candidate for tendon transfers in CP? (2010)
    a. Athetoid
    b. Diplegic
    c. Quadriplegic
A

ANSWER: A

175
Q
  1. All are true about the cervical spine in Down’s syndrome, EXCEPT: (2013, 2016)
    a. Instability occurs only at the atlantoaxial level
    b. Cervical spine xrays have no predictive value of future spine problems
    c. In kids, if the patient is asymptomatic, they do not require screening prior to most sports participation
    d. 25% of Down’s patients have cervical spine problems.
A

ANSWER: A

o Atlantoaxial instability and occiptocervical instability

o Scoliosis (50%)

o Spondylolisthesis

JAAOS 2006 – no screening necessary for asymptomatic downs kids for sports

176
Q
  1. Conditions with Ocular Manifestations: all of the following except (2012)
    a. Homocysteinuria
    b. Marfans
    c. Neurofibromatosis
    d. Achondroplasia
A

ANSWER: D

177
Q
  1. 7yo kid with SED. Planning to do epiphysiodesis for genu valgum. What must you get pre-op? (2015)
    a. C-spine flex/ext
    b. Scoliosis series
    c. MRI brain
    d. Polysomnography
A

ANSWER: A

178
Q
  1. How best to assess limb length discrepancy in a child with a 20deg flexion contracture of the knee: (2015)
    a. Xray scanogram
    b. Measure ASIS to medial malleolus
    c. Blocks
    d. CT scanogram
A

ANSWER: D

179
Q
  1. 28 yo male presents for genetic counselling. Short stature, multiple hand and foot deformities, XRAY shows lytic lesions with calcifications in metacarpals and phalanges. What do you tell him? (2015)
    a. half of his children will be affected
    b. none of his children will be affected
    c. all of his sons and none of his daughters will be affected
    d. all of his children will be affected
A

ANSWER: B

  • Ollier’s disease (multiple enchondromatosis)
  • sporadic inheritance with no genetic predisposition
  • skeletal dysplasia with failure of normal endochondral ossification
  • enchondromas throughout the metaphyses and diaphyses of long bones
  • involved bones are dysplastic, with shortening and bowing
  • risk of malignant transformation <30%
180
Q
  1. Given x-ray of a ball and socket ankle. What is the underlying pathology? (2015)
    a. Ehlers-Danlos
    b. Post traumatic
    c. Lateral ligament instability
    d. Tarsal coalition
A

ANSWER: D

181
Q
  1. What is true regarding the inheritance of x-linked hypophosphatemic rickets? (2016)
    a. If the father has the disease, 50% of his children will get it
    b. If the mother has the disease, 100% of sons will get it
    c. If the father has the disease, 100% of daughters will get it
A

ANSWER: C

182
Q
  1. Larsen’s syndrome. What is common? (2011)
    a. elbow flexion contracture
    b. cervical kyphosis
    c. Sprengel’s deformity
    d. pectus excavatum
A

ANSWER: B

183
Q
  1. All are drugs for systemic CP spasticity except (2009)
    a) Botox
    b) Dantrolene
    c) Baclofen
    d) Clonazepam
A

ANSWER: A

a) presynaptic acetylcholine receptor – acts locally not systemically
b) ryanodine receptor antagonist
c) GABA agonist
d) benzo

184
Q
  1. Congenital Muscular Torticollis. What is true? (2009)
    a) Can palpate tumor
    b) Head turned towards lesion
    c) No known etiology
    d) 50% get DDH
A

ANSWER: A

a) Can palpate tumor – palpate mass
b) Head turned towards lesion – head tilted towards, chin away (head turns away from lesion
c) No known etiology – packaging disorder
d) 50% get DDH – no 5-20%

185
Q
  1. 4 yo child has temp 38 deg, doesn’t want to WB, WBC = 15.5, ESR = 50.1. decreased hip ROM. What do you want to do? (2009)
    a. Bone scan / wbc scan
    b. Admit / start IV abx
    c. Aspirate / possible open hip
    d. Admit for pain control / peds consult
A

ANSWER: C

186
Q
  1. A kid steps on nail at cottage, what is your treatment. (2009)
    a. ID in ER, D/C and Return if any signs of infection
    b. ID in ER and cipro
    c. ID in ER and ceftaz
    d. ID in OR and and iV Abx
A

ANSWER: A

Current recommendations: If barefoot and step in nail – no antibiotics needed, only close followup

187
Q
  1. Pt has a limb length discrepancy. When do you consider lengthening the longer leg? (2009)
    a. To correct varus deformity
    b. Never lengthen long leg
    c. If pelvic obliquity is more significant lengthen the leg to compensate for pelvic obliquity
A

ANSWER: A

188
Q
  1. Jump gait in a CP girl 14 years old (they used the word jump gait specifically). 15 degree hip flexion contracture. 35 degree knee flexion contracture at the knee. -10 degrees of dorsiflexion. Dorsiflexion improved to neutral with knee flexion.
  2. Isolated medial hamstring release
  3. Medial and lateral hamstring release
  4. Medial hamstring and Anterior hemiepiphisiodesis
  5. Medial hamstring and distal femur extension osteotomy
A

ANSWER: D

2019

  • Medial hamstring and distal femur extension osteotomy
  • Deformity
  • Hip: 15 deg
  • Knee: 35 deg
  • Foot: 10 deg PF – neutral with knee flexion
  • Jump gait
  • JAAOS 2014 – Identification of common gait disruption patterns in children with cerebral aplsy
  • Jump gait is characterized by the loss of heel strike at initial contact, with a toe contact pattern for the duration of the stance phase
  • 14 yo F – done growing ie no role for guided grown (eliminates c)
  • Isolated medial hamstring release
  • For mild knee dysfunction, usually younger patients with less than 5 degrees knee flexion deformity (eliminates a)
  • With lateral hamstring – can cause weakness (eliminates b)
  • Medial hamstring + distal femur extension osteotomy
  • Supracondylar femur extension osteotomy +/- patellar tendon advancement or shortening
  • For knee flexion deformities of 10-30 degrees, with severe quadriceps lag close to or already at skeletal maturity
189
Q
  1. Diplegic spastic GMFCs 2 with deterioration in gait. On exam, ankle Dorsiflexion -10 with knee straight, improved to neutral with knee flexion. 3 months post previous botox injection. Improved but has worn off. What is the BEST treatment:
  2. AFO
  3. Repeat botox
  4. Gastrocs recession
  5. Achilles lengthening
A

ANSWER: B vs C

2019

  • UPDATE – Alicia now thinks answer is “Gastroc recession” given positive silverskiold test, and the fact that botox is only a temporary treatment
  • Also since diplegic wouldn’t do bilateral TAL (risk of recurvatum) in an ambulator
  • Age not given…
  • Gastroc recession vs Achilles lengthening
  • JAAOS 2004 – Surgical management of the lower extremity in ambulatory children with cerebral palsy
  • The proposed benefit of Gastroc fascial lengthening techniques is that push-off power is better preserved secondary to sparing of the soleus muscle. There is, however, an increased incidence of recurrent equinus after these procedures.12 Although gait analysis has indicated that gastrocnemius fascial lengthenings improve ankle-power generation at the terminal stance phase for push-off,13 other gait analysis studies have shown equivocal results between patients who had undergone lengthening of the Achilles tendon versus lengthening of the gastrocnemius fascia.14
  • TAL vs gastroc recession
  • rarely indicated as an isolated procedure, except in hemiplegia*** (eliminates c)
  • Botox
  • Competitive inhibitor of presynaptic cholinergic receptors with a finite lifetime (usually lasts 2-3 months) 
  • Used to maintain joint motion during rapid growth when a child is too young for surgery
  • Repeat every 3-6 months – 3 mths too early???
  • AFO
  • JAAOS 2007 – Indications for orthoses to improve gait in children with cerebral palsy
190
Q
  1. 10 year old Langerskiold V blount disease male with severe varus deformity + depression of medial tibial plateau. which one is most unlikely to correct the deformity to anatomic alignment.
  2. Epiphisiolysis of the proximal tibia
  3. Medial plateau elevation
  4. Varus correcting medial tibial osteotomy
  5. Proximal tibial osteotomy
A

ANSWER: A

2019

  • Femoral osteotomy vs epiphysiolysis (doesn’t make sense)
  • 2018 question (different answers???)
  • Distal femoral osteotomy for compensatory valgus deformity
  • Proximal tibial epiphiseolysis (?epiphiseodesis)
  • Varus correcting proximal tibial osteotomy with medial plateau elevation
  • Another proximal tibial osteotomy that sounded more like what we usually do
  • Surgical options for blounts
  • Proximal tibial osteotomy – dome (convex proximal)
  • Growth modulation with tension band
  • Physeal arrest resection and osteotomy – interpose fat of PMMA
  • Hemiplateau elevation – based on arthrogram – medial cresent shaped osteotomy, then ORIF
191
Q
  1. Femoral nerve palsy and pavlik harness disease. All are true except:
  2. Spontaneous resolution of femoral nerve palsy in 60%
  3. Risk factor for surgical management
  4. Can be bilateral
  5. The incidence of femoral nerve palsy is 2.5%
A

ANSWER: A

2019

  • Spontaneous resolution of nerve palsy in 60%
  • Incidence 2.5% (eliminates d)
  • Increased risk of failure of Pavlik (eliminates b)
  • Not commonly bilateral – but can be?
  • Spontaneous resolution – no cases of permanent
  • JBJS 2011 – Femoral Nerve Palsy in Pavlik Harness Treatment for Developmental Dysplasia of the Hip
  • https://oce-ovid-com.proxy1.lib.uwo.ca/article/00004623-201103020-00011/HTML
192
Q
  1. 7 year old kid comes in with 5 week history of painless limp, Trendenlenburg gait. He has symmetric ROM of the hips except for decreased IR and decreased abduction on the affected hip. What is the most likely diagnosis:
  2. DDH
  3. Transient synovitis
  4. Legg-Calve-Perthes
  5. JRA
A

ANSWER: C

2019

LCP

  • Decreased IR, decreased abduction
  • DDH – would not present like this
  • Transient synovitis – painful?
  • JRA – multi joint, more symptoms
  • Perthes
  • Decreased abduction – first to go
  • Decreased internal rotation
  • Early: muscle spasm and synovitis
  • Late: bony impingement
  • Abnormal gait – antalgic/Trendelenberg
193
Q
  1. What is the most common complication of syndactyly:
  2. Digital nerve injury
  3. Vascular injury
  4. Web creep
  5. Wound Issues
A

ANSWER: C

2019

  • Web creep
  • JAAOS 2004 - Surgical Treatment of Congenital Syndactyly of the Hand
  • Web creep – most common complication of surgical treatment (8-60%)
  • The most common complication in syndactyly release is scar formation and progression, either at the webor along the surgical scar (Fig. 10). Web creep is eight times more common with split-thickness skin graft than with FTSG,25 and it is caused by a combination of abnormal tissue, surgical scar, and increasing growth of the underlying osseous structures
  • causes
  • early creep is most commonly caused by necrosis of the tip of the dorsal quadrilateral flap and loss of full-thickness skin graft placed in the web
  • late creep (adolescence) is caused by discrepant growth between scar/skin graft and surrounding tissue during the growth spurt
  • treatment
  • reconstruct web space with local skin flaps
194
Q
  1. Which of the following is false?
  2. Muccopolysaccharidosis have proportional dwarfism
  3. Marfans associated with arachnodactyly
  4. Sanfillipo is the most common
  5. Duchenne is associated with calf hypertrophy (IT DID NOT SAY PSEUDOHYPERTROPHY)
A

ANSWER: D

2019

195
Q
  1. What is true regarding Klippel-Feil syndrome:
  2. Low hairline, webbed neck, decreased neck ROM
  3. Low hairline, short neck, decreased neck ROM
  4. High hairline, short neck, decreased neck ROM
  5. One of them had increased neck ROM
A

ANSWER: B

2019

  • https://rarediseases.info.nih.gov/diseases/10280/klippel-feil-syndrome
    • Klippel Feil syndrome (KFS) is a congenital, musculoskeletal condition characterized by the fusion of at least two vertebrae of the neck. Common symptoms include a short neck, low hairline at the back of the head, and restricted mobility of the upper spine
196
Q
  1. All of the following have ocular problems except:
  2. Marfans
  3. Homocysteinuria
  4. NF
  5. Achondroplasia
A

ANSWER: D

2019

  • Marfans – yes (superior lens dislocation)
  • Homocysteinuria – yes (lens dislocation)
  • NF – yes (Lisch nodules are benign pigmented hamartomas of the iris)
  • Achondroplasia – No
  • ®Homocystinuria
  • -Characterized by tall stature, arachnodactyly, scoliosis, mental retardation, osteoporosis, DVT,
  • and ectopic lens;
  • -Patients w/ clinical features of Marfan’s dz but no family history should have urinalysis for
  • homocystine.
  • -http://www.wheelessonline.com/ortho/marfans_syndrome
  • NF – Optic Glioma, Lisch nodules
  • Marfans – Lens dislocation/subluxation
  • Homocysteinuria – Lens dislocation/subluxation, glaucoma
    • -Marfan – myopia & astigmatism. Risk of (superior) lens dislocation (65%), retinal detachment, early onset cataracts, glaucoma (35%), strabismus.
  • -Neurofibromatosis – Lisch nodules (melanocytic hamartomas on iris, no visual disturbance, pathognomonic for NF-1), plexiform neurofibromas on eyelid (“bag of worms”), choroid hamartomas, retinal tumors (with absence of greater wing of sphenoid & pulsatile proptosis), optic nerve gliomas (15-40%). Gliomas at the chiasm may obstruct 3rd ventricle leading to hydrocephalus.
  • -Achondroplasia – apnea, obesity, recurrent ear infections. NO PROMINENT EYE SYMPTOMS. Risk of blindness prone for spine surgery.
  • -Homocystinuria – lens dislocation (inferior), myopia, glaucoma (similar to Marfan)
197
Q
  1. 8yo child with Bado type II (posterior radial head dislocation – this info was provided in the stem) with an associated PIN palsy. What is the best initial management?
  2. Closed reduction in 100 degrees flexion and supination – type 1 and 3
  3. Closed reduction in 100 degrees flexion and pronation
  4. Closed reduction in extension and pronation
  5. ORIF of ulna and nerve exploration
A

ANSWER: C

2019

  • Rang’s and orthobullets
  • Sagar’s
  • Reduction
  • Type I
  • Achieved with forearm in full supination and longitudinal traction
  • Supination tightens annular ligament – most stable position for PRUJ
  • Then elbow is gently flexed to > 90o to relax biceps
  • Radial head is gently repositioned by direct manual pressure anteriorly on the bone
  • Following reduction, radial head will be stable if left in flexion
  • Type II
  • Traction to forearm with the forearm in full extension
  • Type III
  • Traction to forearm and valgus force over radial head
  • Type IV
  • Depends on fracture characteristics – but likely same as Type I
198
Q
  1. Supracondylar Type III fracture. What is true?
  2. Translation deformity does not remodel
  3. Cross pin and lateral pin are equivalent
  4. Needs emergency surgery
  5. Acceptable to hyperflex to 100 degrees
A

ANSWER: A

2019

  • · Urgent, but not emergent management
  • · The Treatment of Pediatric Supracondylar Humerus Fractures J Am Acad Orthop Surg 2012;20:320-327
  • o 2 lateral pins are clinically but NOT biomechanically equivalent to a crossed-pin construct - 3 lateral pins are equally strong as a crossed-pin construct; medial pin has an increased risk of ulnar nerve injury (NNH = 20)
  • o Translational deformity has minimal remodelling potential and can lead to cubitus varus - the only acceptable deformity is angulation; rotation is also bad
  • o Splinting in hyperflexion increases the risk of compartment syndrome
  • o The AAOS guideline suggests closed reduction with pin fixation for patients with displaced (eg, Gartland types II and III and displaced flexion) pediatric supracondylar fractures of the humerus.
  • o The AAOS guideline suggests the practitioner might use two or three laterally introduced pins to stabilize the reduction of displaced pediatric supracondylar fractures of the humerus. Considerations of potential harm indicate that the physician might avoid the use of a medial pin.
    • Otsuka et al. https://uoftorthopaedics.ca/wp-content/uploads/03.-Basic-paediatrics_Unit-1_Supracondylar.pdf
  • Crossed vs lateral pins
  • crossed pins
  • biomechanically strongest to torsional stress
  • higher risk of ulnar nerve injury (3-8%)
  • highest risk if placed with elbow in hyperflexion as ulnar nerve subluxates anteriorly over medial epicondyle in some children
  • reduce the risk of ulnar nerve injury by
  • placing medial pin with elbow in extension
  • use small medial incision (rather than percutaneous pinning)
  • Angular deformities of the distal humerus are common after supracondylar fractures. The remodeling potential of the distal humerus is limited because the distal physis contributes only 20% of the growth of the humerus. Although remodeling of posterior angulation can occur, angular deformities in the coronal plane will not remodel, resulting in a cubitus varus or cubitus valgus deformity
199
Q
  1. Congenital synostosis. Which of the following is true:
  2. Synostosis is usually middle or distal
  3. Usually presents in supination
  4. Does not improve with resection at the radius
  5. 2 times more common males than females
A

ANSWER: C

2019

  • Miller 5th, 458 - 60% BL - Fixed pronation - Rotational osteotomy at age 5 y.
  • Kozin - Upper-Extremity Congenital Anomalies - JBJS 2003 Radioulnar Synostosis - Associations: Syndactyly or Ulnar Deficiency - Syndromes = Trisomy 21, FAS - Radial Head Dislocation - Fixed Pronation
  • Lovell p 937 - Rare - Usually proximal - M:F = 3:2 - B/L = 80% - Surgery if > 600 fixed pronation - Limited success with excision - Derotation osteotomy = Goal = Fixed Pronation <200 - *** MAJOR COMPLICATION TO R/O = COMPARTMENT SYNDROME post-op ***
200
Q
  1. 10yo child. Intermittent mechanical symptoms. Discoid meniscus question. Which is true?
  2. MRI is more sensitive than physical exam
  3. Surgical management involves saucerization and rim stabilization.
  4. Discoid meniscus is not at higher rate for tearing
A

ANSWER: B

2019

  • JAAOS 2017 - Discoid Lateral Meniscus in Children: Diagnosis, Management, and Outcomes
  • Overall, MRI has a lower sensitivity for diagnosing lateral discoid meniscus than a physical examination.2
  • Current treatment emphasizes meniscal rim preservation with arthroscopic saucerization because of the evidence supporting the effi- cacy of the technique.19,34 The importance of meniscal preservation is highlighted in the work of Baratz et al,3
201
Q
  1. What part of the physis is affected by achondroplasia?
  2. Proliferate zone
  3. Maturation zone
  4. Degenerative
  5. Provisional calcification
A
202
Q
  1. All are true regarding cervical spine in Down Syndrome EXCEPT
  2. Instability occurs only at atlantoaxial level
  3. Cervical spine xray cannot predict progression
  4. If the patient is asymptomatic, they do not require screening prior to most sports participation
  5. 25% of Down patients have cervical spine problems
A

ANSWER: A

2019

  • Also occ-c1
  • JAAOS 2006 - Down Syndrome in Children: The Role of the Orthopaedic Surgeon
  • https://www.ncbi.nlm.nih.gov/pmc/articles/PMC2532127/ - 10-20% AA instability …they quote 18% AAI and 12% AA anomalies
  • https://adc.bmj.com/content/archdischild/early/2018/11/24/archdischild-2018-315751.full.pdf
  • 10-27% c spine involvement but say overestimate
203
Q
  1. Regarding pediatric amputations, all are true except? – Think its all are false except
  2. With all levels of amputation, oxygen consumption is comparable to normal controls because they walk at their own pace.
  3. BKA results in a 60% increase energy expenditure.
  4. Using crutches leads to a 300% increase in energy expenditure.
  5. Double stance is longer on the non-prosthetic limb (pretty sure it said that, but maybe could have said on the prosthetic limb)
A

ANSWER: D

2019

  • With all levels of amputation, oxygen consumption is comparable to normal controls because they walk at their own pace. False – below knee yes but above knee no.
  • BKA results in a 60% increase energy expenditure. False - No increase in kids
  • Using crutches leads to a 300% increase in energy expenditure. Can’t find exact reference but from what I’ve found also False
  • Double stance is longer on the non-prosthetic limb (pretty sure it said that, but maybe could have said on the prosthetic limb) – True. For amputees the relative duration of the stance phase of intact limb, the prosthetic swing phase and the (first) double support phase before the prosthetic single stance phase are significantly increased compared to controls. The (second) double support phase of amputees before the prosthetic swing phase, is shortened but not statistically significant. Compared to the total stance phase, this “second” double stance phase is equal for both controls and amputees (15%).
  • Results: Unilateral transfemoral and hip disarticulation amputations resulted in significantly reduced walking speed
  • (80% and 72% of normal, respectively) and increased VO2 cost (151% and 161% of normal, respectively), while the heart
  • rate was significantly increased in the hip disarticulation group (124% of normal). Compared with the controls, the children with a bilateral amputation walked significantly slower (87% of normal), with an elevated heart rate (119% of normal) but a similar energy cost. Children with a Syme amputation, transtibial amputation, or knee disarticulation walked with essentially the same speed and oxygen cost as did normal children in the same age group.
    • Conclusions: Children with an amputation through the knee or distal to the knee were able to maintain a normal walking
  • speed without significantly increasing their energy cost. Only when the amputation is above the knee do children walk
  • significantly slower and with an increased energy cost.
204
Q
  1. In Early onset scoliosis, all of the following limit thoracic function, EXCEPT:
  2. Proximal fusion to T1 or T2
  3. More Vertebral body levels fused
  4. Not attempting fusion before Age < 12.
  5. T1 to T12 fusion length resulting in thoracic height <18 cm
A

ANSWER: C

2019

205
Q
  1. Pediatric displaced femoral neck fracture, what is true?
  2. Capsule decompression decreases AVN rates
  3. AVN rates are 5%
  4. AVN rates are increased with age <11
  5. It is acceptable to cross the physis with fixation
A

ANSWER: D

2019

206
Q
  1. What is true regarding OM diagnosis
  2. Bone scan is the best test for followup
  3. Xray is good for diagnosis
  4. Sensitivity for a high WBC is low
  5. Sensitivity for a high CRP (>10) is low
A

ANSWER: C (prev B) - Charles

2019

  • The white blood cell count is often normal even in the setting of acute osteomyelitis. The erythrocyte sedimentation rate (ESR) and Creactive protein (CRP) are often elevated; however, they both lack specificity in the absence of other radiologic and microbiologic data. In cases of proven osteomyelitis, both tests may be used to assess response to therapy or relapse. CRP may me more reliable than ESR for assessing response to treatment in children
  • Several imaging modalities are useful in the evaluation of osteomyelitis (Table 1). Plain radiographs are the first step in assessment as they are inexpensive and safe, and may make the diagnosis. Bone destruction and periosteal reaction are not typically seen until infection has been present for 10-21 days.4 Negative films do not exclude the diagnosis of osteomyelitis, especially in acute infection.
207
Q
  1. Displaced Pediatric Supracondylar fracture, all of the following are true EXCEPT: 
  2. Open reduction and fixation of the fracture will still lead to acceptable function and ROM outcomes 
  3. Outcomes are similar to fixing them > 24 hours is equivalent to fixing them emergently 
  4. There is a 20% rate of ulnar nerve injury palsy with a medial pins only 
  5. Cross pinned construct is biomechanically weaker than lateral based pins 
A

ANSWER: C and D

2019

  • Outcomes were satisfactory in 30 (90.9%) patients treated with percutaneous pinning and in 23 (92%) patients treated with open reduction and cross-wiring.
  • The pins are configured to cross proximal to the fracture site in the midline of the distal humerus, and they are advanced through the cortices.2 This configuration has been shown in clinical series to be effective for maintaining reduction and has been shown in biomechanicaltesting21 to be superior to other pin configurations, including multiple lateral entry pins. However, ulnar nerve injury occurs in as many as 10% of patients.2
  • Nerve injury induced by lateral-only pinning occurred at a weighted event rate of 3.4%, while the introduction of a medial pin elicited neurapraxia at a weighted event rate of 4.1%. Lateral pinning carried increased risk of median neuropathy, whereas the use of a medial pin significantly increased the risk of ulnar nerve injury.
  • Delay in surgery did not result in an increased rate of major complications following closed reduction and percutaneous pinning of type II supracondylar humerus fractures in children. Further prospective work is necessary to determine if there are subtle treatment benefits from emergent treatment of type II supracondylar humerus fractures.
208
Q
  1. what is not an appropriate option to treat elbow contracture in an arthrogrypotic patient? 
  2. Anterior deltoid transfer 
  3. Pec transfer to biceps 
  4. Steindler Flexoroplasty 
  5. Triceps to biceps 
A

ANSWER: A

2019

can do triceps transfer BUT not all of it

  • Transfer of the entire triceps to the biceps should not be done in the amyoplasia child. Unopposed force across any joint will result in a contracture, and fixed flexion is much worse than a supple joint with passive flexion. There is no salvage procedure for a rigid, fixed elbow flexion contracture in these children. Bimanual function is impeded and this procedure makes the child worse. The Steindler flexorplasty, in which the flexor-pronator muscle origin is relocated to a position more anterior and more proximal on the distal humerus depends for function on a strong wrist extensor to counterbalance wrist flexion and transfer excursion to the elbow. Children with amyoplasia do not have this strong wrist extensor, and this procedure may worsen the wrist flexion deformity.10 The Clark transfer, in which the insertion of the pectoralis major muscle is prolonged with a tendon graft that inserts into the biceps tendon, has the wrong line of pull and results in pure shoulder adduction. Bipolar transfers of either the pectoralis major muscle or the latissimus muscle are potential options to restore active elbow flexion. Both rely on the presence of a good-quality muscle and proper tensioning of the transfer.
  • Amyoplasia is a condition characterized by a generalized lack in the newborn of muscular development and growth, with contracture and deformity at most joints. It is the most common form of arthrogryposis
209
Q
  1. Tri-plane fractures of the ankle, what is true? 
  2. SH4 on the lateral view 
  3. SH4 on the coronal 
  4. SH2 on the lateral, and SH3 on the coronal 
  5. SH3 on the lateral, and SH2 on the coronal 
A

ANSWER: C

2019

210
Q
  1. What is the best indicator of peak growth velocity in pediatric growth? 
  2. Closure of the olecranon apophysis 
  3. Tri-radiate closure 
  4. Risser 1 
  5. Menarche 
A

ANSWER: A

2019

The morphological development of the olecranon represents a simple but reliable method of skeletal age assessment during the phase of peak height velocity.

211
Q
  1. With respect to Duchenne muscular dystrophy, which of the following delays muscle atrophy/weakening? 
  2. NSAIDs 
  3. Prednisone 
  4. Bisphosphonates 
  5. Something else that was wrong 
A

ANSWER: B

2019

  • Corticosteroids, including prednisone and a related compound, deflazacort, have recently been shown to markedly delay the loss of muscle strength and function in boys with Duchenne muscular dystrophy.
212
Q
  1. Tibial spine fracture in pediatrics. What is the most associated with arthrofibrosis? 
  2. Screw vs. suture repair 
  3. Length of post-op immobilization 
  4. Open vs arthroscopic surgery 
  5. Associated meniscal tear 
A

ANSWER: B

2019

213
Q
  1. Regarding bisphosphonates given in a pediatric patients, all of these are true except: 
  2. Causes increased sclerosis below the physis (they actually said “below” the physis but whatever – they were describing Harris growth arrest lines) 
  3. Poor bioavailability when taken orally? 
  4. Bisphosphonates causes growth retardation 
  5. The molecules of bisphosphonates remain in bone for years 
A

ANSWER: C

2019

214
Q
  1. 11 yo female gymnast with wrist pain - what would you expect to find on XRAY?
  2. Postive ulnar variance
  3. Decreased radial inclination
  4. Negative ulnar variance
  5. CMC arthritist
A

Answer: C (2017)

A – True – premature closure from physeal injury

B - no

C - early on yes- risk factor

D – not related

215
Q
  1. A 2 year old male has rhizomelic dwarfism, frontal bossing and midface hypoplasia. He presents due to his parents complaining of loud snoring. The results of a sleep study on the chart show he is suffering from central apnea. Which is the next best investigation?
  2. MRI for C1-2 instability
  3. Serum CK
  4. MRI for foramen magnum stenosis
  5. Urine muccopolysaccarides
A

Answer: C (2017)

A - not in achondroplasia

B - no
C - yes

D – not the right features, proportionate dwarfism.

  • foramen magnum stenosis
  • may cause periods of apnea or suddent death in infants
    • Rhizomelic (proximal limb shortening)
  • hypochondroplasia
  • achondroplasia
  • chondrodysplasia punctata
  • pseudoachondroplasia 
  • thanatophoric dysplasia
  • particularly type II
  • kyphomelic dysplasias
  • Mesomelic (middle segment limb shortening)
  • dyschondrosteosis (Leri-Weil disease): limb shortening with a Madelung deformity
  • mesomelic dysplasia
  • type Langer
  • type Reinhardt-Pfeiffer
  • Acromesomelic (middle and distal segment limb shortening)
  • chondroectodermal dysplasia - Ellis-van Creveld syndrome 
  • similar to asphyxiating thoracic dysplasia but 
  • (a) hexadactyly is a constant finding
  • (b) there is severe hypoplasia of the fingers and nails
  • (c) congenital heart disease is common and 
  • (d) hypoplastic lateral tibial plateau is characteristic in childhood
  • acromesomelic dysplasia
  • mesomelic dysplasia
  • type Nievergelt
  • type Robinow
  • type Werner
  • Acromelic (distal segment shortening)
  • asphyxiating thoracic dysplasia/Jeune’s syndrome: 
  • narrow thorax with short ribs leading to respiratory distress
  • spur-like projections of the acetabular roof. Premature ossification of the femoral capital epiphyses
  • occasional postaxial hexadactyly
  • cone-shaped epiphyses in childhood
  • peripheral dysostosis
216
Q
  1. What lower extremity alignment most predisposes to lateral patella dislocations?
  2. Femoral anteversion with external tibial torsion
  3. Metatarsus adductus and internal tibial torsion
  4. Femoral anteversion with metatarsus abductus
  5. Femoral retroversion with external tibial torsion
A

Answer: A (2017)

A - yes

B – no metatarsus adductus and internal is wrong

C – anteversion right, adductus is wrong

D – retroversion is wrong, external tibia is right

  • Miserable malalignment syndrome: External tibial torsion, femoral anteversion, genu valgus
    • a term named for the 3 anatomic characteristics that lead to an increased Q angle 
  • femoral anteversion
  • genu valgum
  • external tibial torsion / pronated feet
217
Q
  1. A 7yo female has a medial epicondyle fracture that is displaced 5 mm and is treated nonopoperatively. At 12 weeks she is pain free and functioning well but XR suggests fibrous union. What do you do next?
  2. Re-cast for another 6 weeks
  3. ORIF
  4. organize a f/u in 4-6 months
  5. MRI to assess for union
A

Answer: C (2017)

A – no more cast needed

B – fibrous is okay

C- true

D – fibrous is okay

218
Q
  1. 11yo male, previous leukemia has had full body radiation and Bone marrow transplant. He is currently in remission. He has had a three month history of a limp and mild right hip pain. They show and AP an frog leg lateral with a R hip mild, chronic appearing SCFE. What is the next best step?
  2. Bilateral hip MRI to rule out AVN
  3. In situ pinning of bilateral hips
  4. Local and systemic staging
  5. Abduction brace
A

Answer: B (2017) – pin in place

219
Q
  1. 8-year-old boy with distal radius fracture. He is treated with a closed reduction and casting. 10 days following the injury, he shows up in clinic with his fracture dorsally angulated by 20 degrees. His cast remains well molded and well fitting. What should you do?
  2. observe and remove cast in 4-5wks
  3. Remanipulate
  4. Perform drill osteoclasis, correction of the dorsal angulation and fixation
  5. Change the cast
A

Answer: A (2017)

Up to 30 degrees is acceptable

220
Q
  1. A 14 year old male with cerebral palsy is able to walk short distances with 2 crutches at home, and long distances with 2 crutches at school. When travelling longer distances at the mall, he requires the use of a wheelchair. What is his score on the Gross Motor Functioning Classification System?
  2. GMFCS I
  3. GMFCS II
  4. GMFCS III
  5. GMFCS IV
A

Answer: C (2017)

221
Q
  1. 14.5-year-old boy with 4cm LLD. 2cm from the tibia and 2cm from the femur. What is the best option?
  2. Perform immediate contralateral distal femoral and proximal tibial epiphysiodesis
  3. Delayed contralateral distal femoral and proximal tibial hemiepiphysiodesis
  4. Contralateral immediate distal femoral hemiepiphysiodesis
  5. Observe and plan for lengthening at skeletal maturity
A

Answer: A (2017)

Need to make up a lot. Has until 16 yrs old.

222
Q
  1. Kid with multiple fractures including olecranon fractures. What do you to recommend?
  2. Work-up for NAI
  3. Intravenous Bisphosphonates Therapy
  4. Biopsy of the fracture site
  5. Genetic testing to rule out Collagen Type II disorder
A

Answers: B (2017) – OI, type 1 collagen disorder… would work up for NAI, dx OI and then treat with Bisphosphonates. Prob poorly remembere

223
Q
  1. Child with flexible pes planus. What is most likely?
  2. Good relief with orthotics alone
  3. Will need soft tissue releases
  4. Tight heel cord
  5. Will not improve without surgical intervention
A

Answer: C (2017)

  • 25% are associated with gastrocnemius-soleus contracture
  • The best treatment for this condition is taking enough time to convince the family that it will not cause any problems and no treatment is necessary. For the occasional symptomatic patient or the child whose parents continue to demand treatment, some inexpensive modality, such as off-the-shelf arch supports, can be tried.
224
Q
  1. 4 year old child with idiopathic toe walking. Which of the following statements is true?
  2. Most cases resolve spontaneously
  3. Is not associated with tight heel cord
  4. Is associated with hyperextension of knee during gait
  5. The child most likely has a leg length discrepancy
A

Answer: A (2017)

A - true

B – false, it is associated.

C – false – nope, its flexion

D – false. Not if bilateral.

225
Q
  1. You are consulted to see a newborn with a foot deformity. They provide an XR of a patient with posteromedial bowing of the tibia and associated calcaneovalgus foot. What will you advise this patient’s family?
  2. They will require serial casting and surgical treatment
  3. This is associated with a genetic abnormality
  4. They will require early soft tissue releases
  5. Long-term orthopaedic follow up should be arranged for an expected limb length discrepancy
A

Answer: D (2017), normal bowing

226
Q
  1. 8M with cerebral palsy has a 45-degree flexion contracture in an elbow that has good function. What is contra-indicated?
  2. Musculocutaneous neurectomy
  3. Brachilais lengthening
  4. Biceps lengthening
  5. Flexor pronator advancement
A

Answer: D (2017) - (Prev A) Charles says D – this is for MCN palsy….why TF would you do this for flexion contracture. Would also create unbalanced flexion force in spastic condition (CP).

A – described in the 1980’s. But only for non-functional limb

B - described

C – described.

D – prob make it worse. This is done in Musculocutaneous nerve palsy to regain elbow flexion. IF you move it distal then it’s okay.

227
Q
  1. What is true of Duchenne muscular dystrophy
  2. Autosomal dominant and an issue with the gene for dystrophen
  3. Autosomal dominant and an issue with the gene coding for Fukutin
  4. X-linked and an issue with gene coding for dystrophen
  5. X-linked and an issue with gene coding for fukutin
A

ANSWER: C

2018

228
Q
  1. 18 year old female has knee pain. You’re shown multiple coronal MRI cuts of what I thought was a discoid meniscus with a tear / increased signal. What is the best treatment plan?
  2. Complete menisectomy
  3. Partial menisectomy and ensure the meniscus is stable
  4. Meniscus transplant
  5. Repair of meniscal tear
A

ANSWER: B

2018

best available answer – read exam carefully as discoid should be treated with discoid meniscus saucerization + meniscal repair

229
Q
  1. In pediatric infections, when should you send cultures for anaerobe, TB, and fungus in addition to the usual aerobe cultures?
  2. This should be done in all pediatric patients
  3. When there is pyomyositis
  4. When there is associated osteomyelitis
  5. When the child is <19 years old
A

ANSWER: B

2018

  • there are fungal pyo and TB pyo
  • pyo considered tropical disease
  • only choice that makes sense
230
Q
  1. A 10 year old kid has blounts. He is Lengenskoild V and has depression of the medial tibial plateau. Which of the following surgical options would be LEAST helpful?
  2. Distal femoral osteotomy for compensatory valgus deformity
  3. Proximal tibial epiphiseolysis (?epiphiseodesis)
  4. Varus correcting proximal tibial osteotomy with medial plateau elevation
  5. Another proximal tibial osteotomy that sounded more like what we usually do
A

ANSWER: A

We’re thinking that Blount’s kids have varus femurs as well - correcting the varus would be good. Poorly worded answers it appears.

2018

Most helpful would be C

LEAST helpful would be proximal tibial epiphyseodesis - doesn’t address any malalignment

231
Q
  1. All of the following are associated with idiopathic toe walking EXCEPT?
  2. Decreased passive dorsiflexion
  3. Increased tone in upper extremities
  4. Autism
  5. Learning disability
A

ANSWER: B

2018

  • Causes: CP, Idiopathic (this question), Duchennes etc
  • Idiopathic toe walking is often associated with Autism Spectrum Disorders, ADHD, Specific Learning Disabilities, Developmental Delays and other disabilities with sensory processing difficulties, but can occur with typical children, as well.
  • main causes of toe walking are idiopathic toe walking and cerebral palsy (orthobullets)
232
Q
  1. All of the following are true of pediatric avulsion fractures EXCEPT?
  2. Iliac avulsions are often treated non op
  3. Tibial tubercle avulsions usually need a CT or MRI for diagnosis
  4. The growth plate is the weakest part of the tendon attachment
  5. Olecranon avulsion associated with OI
A

ANSWER: B

2018

233
Q
  1. What is the correct order of the regions of the growth plate, starting at the metaphysis and moving to the articular surface?
  2. Hypertrophic, proliferative, reserve
  3. Proliferative, hypertrophic, reserve
  4. Reserve, proliferative, hypertrophic
  5. Hypertrophic, reserve, proliferative
A

ANSWER: A

2018

234
Q
  1. A 12 year old girl has ongoing ankle pain. Her developmental milestones were normal and there is no family history of anything significant. You are shown a clinical photo of a ball and socket ankle. What is the most likely cause of her pain?
  2. Tarsal coalition
  3. clubfoot
  4. Ehlers danlos
A

ANSWER: A

2018

235
Q
  1. What is true regarding CRMO
  2. After core biopsy it is best treated with irrigation an debridement followed by antibiotics
  3. A bone scan is needed in the work up
  4. It is an inflammatory condition that behaves like osteomyelitis but has sterile cultures
A

ANSWER: C

2018

Treated with NSAID and pamidronate

236
Q
  1. There is a 10 year old boy. He has recurrent patella instability and a TTTG measuring 26mm. Which of the following is the LEAST recommended course of action
  2. Medial patellofemoral soft tissue reconstruction with hamstring (?or quad?) graft
  3. Supra-patellar soft tissue realignment procedure
  4. Tibial tubercle osteotomy
  5. ?Medial soft tissue tenodesis of some sort
A

ANSWER: C

2018

*Growth plate there…don’t be dumb

Instead do Roux-Goldthwait

237
Q
  1. What is the most common position of a clubfoot deformity?
  2. Forefoot pronation, midfoot adductus, hindfoot inversion, ankle equinus
  3. Forefoot pronation, midfoot abduction, hindfoot eversion, ankle equinus
  4. Forefoot supination, midfoot adductus, hindfoot inversion, ankle equinus
  5. Forfoot neutral/supination, midfoot adductus, hindfoot neutral, ankle neutral
A

ANSWER: A

2018

  • JAAOS 2003 – Nonsurgical management of idiopathic clubfoot
  • At birth, the idiopathic clubfoot is severely supinated, but the forefoot is still adducted and pronated relative to the hindfoot, which is in varus and in equinus.
  • The goal of the initial cast is to align the forefoot with the hindfoot. Ponseti describes the forefoot as pronated in relation to the hindfoot, so supinating the forefoot and elevating the first metatarsal improves this alignment.
238
Q
  1. A 3 year old child was treated as a baby with ponsetti casing and perc heel tenotomy for club foot. He now presents to you. When he walks, he persistently strikes the lateral aspect of his foot on the floor and has hindfoot varus. What is the best INITIAL treatment?
  2. Repeat ponseti casting
  3. Tib ant transfer
  4. Tib ant transfer and lateral midfoot closing osteotomy
A

ANSWER: A

2018

239
Q
  1. Which of the following is an indication for a CLO?
  2. A child with a flatfoot and ill defined pain with activity
  3. A child with an incongruent TN joint (?and something else)
  4. A child with a tight heel cord and a flexible flatfoot that has failed non surgical treatment
  5. Overcorrection of a clubfoot with a congruent TN joint and a normal thigh foot angle
A

ANSWER: B

2018

240
Q
  1. All of the following are true regarding Syme amputations in children EXCEPT:
  2. Heel pad migration is a common problem
  3. If you leave the calcaneus attached, the heel pad will grow as the child grows
  4. Children function as a level equal to their peers
A

ANSWER: B

2018

241
Q
  1. Regarding entirely metaphyseal distal radius fractures in children treated with closed reduction and casting with of the following is TRUE about re-displacement
  2. Happens in 30%
  3. More common if younger than 10
  4. An above elbow cast prevents re-displacement
  5. Transverse patterns are more unstable
A

ANSWER: A

2018

    • more common >10yrs
    • oblique more unstable
  • Orthobullets
  • After failed initial treatment with closed reduction and casting, displaced distal radius/forearm fractures should be treated with repeat closed reduction. Percutaneous fixation can decrease the risk of re-displacement.
  • Approximately 20-25% of distal radius metaphyseal fractures re-displace early after closed reduction and casting. Significant displacement can lead to poor functional and cosmetic outcomes. Indications for surgical fixation of distal forearm fractures in adolescents >10 years old include: angulation >20 degrees and rotation >30 degrees.
  • McLauchlan et al., in their prospective randomized, controlled trial, showed that K-wire fixation and casting was superior to closed manipulation (MUA) and casting for displaced distal radius fractures when comparing outcomes of loss of reduction and number of radiographs. Of the 33 patients in the MUA group, seven had to undergo a second procedure to correct recurrent displacement, compared to one of the 35 in the K-wire group who required exploration for recovery of a migrated pin.
  • Kamat et al. present a retrospective review of 1001 pediatric distal forearm fractures that required closed reduction to identify factors associated with re-displacement. They found the cast index (CI) - or ratio of sagittal (lateral) and coronal (AP) inner cast diameters - to be the only significant predictor. CI > 0.8 was associated with a significantly higher rate of displacement (26%) than CI <= 0.8 (5.6%). Previous studies have reported an ideal CI of 0.7.
    • Bae et al. [7] defined distal radius fractures as “acceptable” in a bayonet apposition; specifically, in the sagittal plane up to 30° if more than five years of growth is remaining and 5° less for each year less than five, and in the frontal plane 10–15° if more than five years of growth is remaining.
  • Recently, Alemdaroglu et al. [6] introduced a new factor; they found that complete initial displacement of the fracture and increased obliquity of the fracture line were the most important risk factors for re-displacement
  • https://www.ncbi.nlm.nih.gov/pmc/articles/PMC3385893/pdf/264_2012_Article_1493.pdf
  • A wide range of re-displacement rates has been reported in the literature, ranging from 24% to 45.8% [9–11, 14, 15]. A 35% re-displacement rate was found in our series, with a higher re-displacement rate within the older cohort.
  • Age (14+ vs 0–13) 4.8 (1.6,14.1) 0.01
  • PR translation (lat)(10% vs 0–9%) 4 (1.3,11.9) 0.03
242
Q
  1. 14-year-old female has a proximal humerus fracture. It is 60% displaced and 45 degrees angulated. How should you manage it?
  2. Open reduction
  3. CRPP
  4. Hanging arm cast
  5. Abduction arm/chest cast
A

ANSWER: B

2018

  • Orthobullets - >12 years old = up to 45° of angulation or 2/3 displacement
  • JAAOS Feb 2015 - >13 yrs – lower chance of remodelling – consider surg for >2/3 displacement
  • can argue that 60% is less then 2/3 but lets say more then 45 degrees angulated
  • Nonoperative
  • immobilization
  • indications
  • acceptable alignment for non-operative management
  • <10 years old = any degree of angulation
  • 10-12 years old = up to 60-75° of angulation
  • >12 years old = up to 45° of angulation or 2/3 displacement
  • technique
  • immobilization modalities
  • sling +/- swathe
  • shoulder immobilizer
  • coaptation splint
  • Operative
  • closed reduction +/- fixation
  • indications
  • unacceptable alignment for non-operative management as described above
  • open reduction internal fixation
  • indications
  • unable to obtain acceptable reduction due to soft tissue interposition
  • long head of biceps tendon (most common)
  • joint capsule
  • infolded periosteum
  • deltoid muscle
  • open fractures
  • fractures associated with vascular injuries
  • intra-articular displacement
243
Q
  1. All of the following regarding fishtail deformities of the distal humerus are true EXCEPT?
  2. Represents a central growth arrest of the distal humerus
  3. Can occur following distal humerus fracture
  4. Can lead to ulnohumeral arthritis
  5. Contributes to a large limb length discrepancy
A

ANSWER: D

2018

Lateral trochlear epiphysis growth arrest

JAAOS 2019 - Current Concepts in the Treatment of Lateral Condyle Fractures in Children - lateral trochlear ossification center fails to develop because of disrupted blood supply to the trochlea

244
Q
  1. 15 year old boy playing basketball lands and gets tibial tubercle avulsion. He should be evaluated closely for
  2. ispilateral tibial eminence fracture
  3. contralateral Osgood-Schlatter
  4. compartment syndrome
  5. Osteogenesis imperfecta
A

Answer: C (2017) compartment syndrome